You are on page 1of 62

CURRENT 3.

Current Electricity

Section A - Definition of Current, Current 2. ELECTRIC CUR R ENT AND


Density, Drift Velocity
CURRENT DENSITY
1. INTRODUCTION When there is a transfer of charge from one side
• Conductor of an area to the other, we say that there is an
electric current through the area. If the moving
In some materials, the outer electrons of each atoms
charges are positive, the current is in the direction
or molecules are only weakly bound to it. These
of motion, if they are negative, the current is
electrons are almost free to move throughout the
opposite to the direction of motion. If a charge Q
body of the material and are called free electrons.
crosses an area in time t, we define the average
They are also known as conduction electrons. When
electric current through the area during this time as
such a material is placed in an electric field, the
free electrons move in a direction opposite to the Q
i
t
field. Such materials are called conductors.
• Insulator : The current at time t is

Another class of materials is called insulators in Q dQ


i  lim 
which all the electrons are tightly bound to their t 0 t dt

respective atoms or molecules. Effectively, there Thus, electric current through an area is the rate of
are no free electrons. When such a material is transfer of charge from one side of the area to the
placed in an electric field, the electrons may slightly other. The SI unit of current is ampere. If one
shift opposite to the field but they can't leave their coulomb of charge crosses an area in one second,
parent atoms or molecules and hence can't move the current is one ampere. It is one of the seven
through long distances. Such materials are also base units accepted in SI.
called dielectrics.
• Semiconductor : EXAMPLE 1

In semiconductors, the behaviour is like an insulator If q = 2t2 + 3, find current at t = 2 sec ?


at low levels of temperature. But at higher dq
Sol. i
temperatures, a small number of electrons are able dt
to free themselves and they respond to the applied i = 4t
electric field. As the number of free electrons in a  i at 2 sec = 4 × 2 = 8 A
semiconductor is much smaller then that in a We shall now define a vector quantity known as
conductor, its behaviour is in between a conductor electric current density at a point. To define the
and an insulator and hence, the name semiconductor. current density at a point P, we draw a small area
A freed electron in a semiconductor leaves a S through P perpendicular to the flow of
vacancy in its normal bound position. These charges(shown in figure) If i be the current through
vacancies also help in conduction. the area S, the average current density is
 i For a finite area,
j   
S i   j.dS
The current density at the point P is
Note carefully that an electric current has direction
i di as well as magnitude but it is not a vector quantity.
j  lim 
S0 S dS It does not add like vectors. Therefore current is
neither a vector quantity nor a scalar quantity but a
The direction of the current density is the same as
tensor quantity. The current density is a vector
the direction of the current. Thus, it is along the
quantity.
motion of the moving charges, if the charges are
positive and opposite to the motion of the charges,
if the charges are negative. If a current i is uniformly EXAMPLE 2
distributed over an area S and is perpendicular to it, An electron beam has an aperature 1.0 mm2. A
i total of 6.0 × 10 10 electrons go through any
j
S perpendicular cross-section per second. Find (a) the
current and (b) the current density in the beam.
Sol. The total charge crossing a perpendicular cross-
Q section in one second is
P q = ne
= 6.0 × 1016 × 1.6 × 10–19 C
Q S = 9.6 × 10–3 C
 i
t The current is
(a)
q 9.6  10 –3 C
i  = 9.6 × 10–3 A
t 1s

As the charge is negative, the current is opposite to


S cos 
 be direction of motion of the beam.
j

(b) The current density is

S n̂ i 9.6  10 –3 A 9.6  10 –3 A
j  
S (1.0mm)2 1.0  10 –6 m2
Now let us consider an area S which is not
necessarily perpendicular to the current (figure shown)  9.6  103 A / m 2
If the normal to the area makes an angle  with the
direction of the current, the current density is,
3. DRIFT SPEED
i A conductor contains a large number of loosely
j
Scos  bound electrons which we call free electrons or
or, i = j S cos  conduction electrons. The remaining material is a
 collection of relatively heavy positive ions which
where i is the current through S, If S be the we call lattice. These ions keep on vibrating about
area vector corresponding to the area S, we have their mean positions. The average amplitude
  depends on the temperature. Occasionally, a free
i  j.S
electron collides or interacts in some other fashion
with the lattice.
The speed and direction of the electron changes
e
randomly at each such event. As a result, the electron vd  E  kE ...(1)
2m
moves in a zig-zag path. As there is a large number
of free electrons moving in random directions, the opposite to the field. The constant k depends on the
number of electrons crossing an area S from one material of the conductor and its temperature.
side very nearly equals the number crossing from
E
the other side in any given time interval. The electric A vd
current through the area is, therefore, zero.
v d t
When there is an electric field inside the conductor,
a force acts on each electron in the direction Let us now find the relation between the current
opposite to the field. The electrons get biased in density and the drift speed. Consider a cylindrical
their random motion in favour of the force. As a conductor of cross-sectional area A in which an
result, the electrons drift slowly in this direction. At electric field E exists. Consider a length vd t of the
each collision, the electron starts afresh in a random conductor (figure shown). The volume of this
direction with a random speed but gains an additional portion is Avdt. If there are n free electrons per
velocity v' due to the electric field. This velocity v' unit volume of the wire, the number of free electrons
increases with time and suddenly becomes zero as in this portion is nAvdt. All these electrons cross
the electron makes a collision with the lattice and the area A in time t. Thus, the charge crossing this
starts afresh with a random velocity. As. the time ,t area in time t is
between successive collisions is small, the electron
Q = nAvd t e
"slowly and steadily drifts opposite to the applied
field (shown figure). If the electron drifts a distance Q
or, i = nAv
vde
 in a long time t, we define drift speed as t

l i
vd  and j=  nev d ...(2)
t A
If  be the average time between successive
collisions, the distance drifted during this period is
EXAMPLE 3
1 1  eE  Calculate the drift speed of the electrons when 1 A
l  ( ) 2    ( ) 2
2 2 m  of current exists in a copper wire of cross-section 2
mm2. The number of free electrons in 1 cm3 of
l 1  eE  copper is 8.5 × 1022.
The drift speed is v d    
 2 m  Sol. We have
It is proportional to the electric field E and to the j i
average collision-time . j = nevd or, vd  
ne A ne
The random motion of free electrons does not
contribute to the drift of these electrons. Also, the 1A
average collision-time is constant for a given = (2  10 –6 m 2 )(8.5  1022  106 m –3 )(1.6  10 –19 C)
material at a given temperature. We, therefore,
make the following assumption for our present = 0.036 mm/s
purpose of discussing electric current. We see that the drift speed is indeed small.
When no electric field exists in a conductor, the
Note
free electrons stay at rest (Vd = 0) and when a
field E exists, they move with a constant velocity
Section B - Resistance & resistivity, Ohm's R is called the resistance of the given conductor.
law + VI (Volt - Curr.) The quantity 1/R is called conductance.
Characteristics + Colour Code
+ Temp. Dependence Equation (5) is another form of Ohm's law which is
widely used in circuit analysis. The unit of resistance
4. OHM'S LAW
is called ohm and is denoted by symbol . An object
Using equations (1) and (2)
of conducting material, having a resistance of desired
ne 2  value, is called a resistor.
j  nev d  E or, j  E ...(3)
2m From equation (5) and (6)

ne 2  
where  = R ...(7)
2m A
where  depends only on material of the conductor From equation (7), the unit of resistivity  is ohm-
and its temperature. This constant is called the metre, also written as -m. The unit of conductivity
electrical conductivity of the material. Equation (3) () is (ohm-m) written as mho/m.
is known as Ohm's law.
from eq. (4) & (7)
The resistivity of a material is defined as
1 2m 2 m  
  R 
...(4) ne2 A A
 ne 2 
Ohm's law tells us that the conductivity (or
 2m 
resistivity) of a material is independent of the electric where  = resistivity  where  
field existing in the material. This is valid for  ne 2  
conductors over a wide range of field.
 = length along the direction of current
Suppose we have a conductor of length l and
uniform cross-sectional area A (figure shown) Let A = Area of the cross section perpendicular
us apply a potential difference V between the ends to direction of current
of the conductor. The electric field inside the n = no. of free charges per unit volume.
V  = relaxation time
conductor is E  . If the current in the conductor
l m = mass of electron
i
is i, the current density is j  . Ohm's law j = E
A
EXAMPLE 4
then becomes
Calculate the resistance of an aluminium wire of
v length 50 cm and cross-sectional area 2.0 mm2. The
resistivity of aluminium is  = 2.6 × 10–8 -m
i
A
E 
Sol. The resistance is R = 
l A
(a) (b)
(2.6  10 –8   m)  (0.50m)
  0.0065 
i V 2  10 –6 m 2

A l We arrived at Ohm's law by making several
1 l l assumptions about the existence and behaviour of
or, V i i ...(5) the free electrons. These assumption are not valid
A A
for semiconductors, insulators, solutions etc. Ohm's
or, V=Ri ...(6)
law cannot be applied in such cases.
EXAMPLE 5
L2 L I 
The dimensions of a conductor of specific resistance  {r1L  (r2 – r1 )x}–1   – 
 0
 (r2 – r1 ) 
 are shown below. Find the resistance of the
conductor across AB, CD and EF.
– L  1 1  L
A D   – 
(r2 – r1 )  r2 r1  (r1r2 )

c F IL
E  V  IR 
b r1r2
a
C
B
EXAMPLE 7
c b a
Sol. R AB  , R CD  , R EF  The space between two coaxial cylinders, whose
ab ac bc
radii are a and b (where a < b as in (figure shown)
is filled with a conducting medium. The specific
EXAMPLE 6 conductivity of the medium is .
A portion of length L is cut out of a conical solid

a
wire. The two ends of this portion have circular
b
cross-sections of radii r 1 and r2 (r2 > r1). It is
connected lengthwise to a circuit and a current i is
flowing in it. The resistivity of the material of the
wire is . Calculate the resistance of the considered
portion and the voltage developed across it.
Sol. If follows from the figure, that

r2 – r1 (a) Compute the resistance along the length of


tan = cylinder.
L
 r = r1 + x tan  (b) Compute the resistance between the cylinders
in the radial direction. Assume that the cylinders
 r2 – r1  r1L  x(r2 – r1 ) are very long as compared to their radii, i.e., L >>
= r1 + x  
 L  L
b, where L is the length of the cylinders.

l l l l
i r2 Sol. (a) R =   
r1 A A ( b 2 – a 2 ) (b2 – a 2 )
l (b) From Ohm's law, we have
 
J  E
 
 A  r 2  [r1L  (r2 – r1 )x]2 Assuming radial current density. J becomes
L2
 I
dx dxL2 J rˆ for a < r < b
dR  2
 2 rL
r [r1L  (r2 – r1 )x]2
 I
L2
L
dx and, therefore, E rˆ
R   dR  2rL
 0 [r1L  (r2 – r1 )x]2

Here we have used the assumption that L >> b so Colour band A and B : Indicate the first two
  significant figures of resistance in ohm.
that E and J are in cylindrically symmetric form.
Band C : Indicates the decimal multiplier i.e. the
The potential drop across the medium is thus : number of zeros that follows the two significant
a  a
figures A and B.
 I dr I b
Vab    E(r).dr = –  ln   Band D : Indicates the tolerance in percent about
2 L b r 2 L  a 
b the indicated value or in other words it represents
The resistance the percentage accuracy of the indicated value.
The tolerance in the case of gold is ± 5% and in
b silver is ± 10%. If only three bands are marked on
In  
Vab a carbon resistance, then it indicate a tolerance of 20%.
R ab    
I 2 L Table : Colour code for carbon resistance

Method 2 : We split the medium into differential cylindrical Letters as an Figure Multiplier
shell elements of width dr, in series. The current aid to Colour
(A, B) (C)
memory
flow is cylindrically symmetric (L >> b). The area
B B lack 0 10 o
through which the current flows across a shell of
radius r is A(r) = 2rL. The length the current flows, B B rown 1 10 1

passing through a shell of radius r is dr. Therefore, R Red 2 10 2

the resistance of the shell of radius r is : O Orange 3 10 3


Y Y ellow 4 10 4
1 dr Green 5 10 5
dR  G
 2rL
B Blue 6 10 6
Since the shells are connected in a series, we have V Violet 7 10 7
G Grey 8 10 8
 b W White 9 10 9
b ln  
a
R ab   dR   
a
2  L Ex.1 A voltage of 30 V is applied across a carbon resistor
with first, second and third rings of blue, black and
yellow colours respectively. Find the value of
Colour Coding of Resistance current through the resistor.
To know the value of resistance colour code is used. Sol. Blue Black yellow
These code are printed in form of set of rings or   
strips. By reading the values of colour bands, we 6 0 4
(A) (B) (C)
can estimate the value of resistance.
 R = AB × 10C = 6 × 104 ~
The carbon resistance has normally four coloured
V 30
rings or bands say A, B, C and D as shown in  i= = = 5 × 10–5A
R 60 10 4
following figure.

Ex. 2 A voltage of 30 V is applied across a carbon resistor


with first, second and third rings of blue, black and
A B C D red colours respectively. Find the value of current
through the resistor.
Sol. Blue Black Red
  
6 0 2
(A) (B) (C)
 R = AB × 10C = 6 × 102 ~ (c) Resistance of semiconductors
V 30 (i) There are certain substances whose conductivity
 i= = = 5 × 10–2A
R 60 10 2 lies in between that of insulators and conductors,
higher than that of insulators but lower than that of
conductors. These are called as semiconductors,
e.g., silicon, germanium, carbon etc.
Effect of Temperature on Resistance (ii) The resistivity of semiconductors decreases with
(a) Resistance of Pure Metals increase in temperature i.e.,  for semiconductors
is –ve and high.
 2m  l (iii) Though at ordinary temperature the value of n
(i) We know that R 2 
 ne   A (no. of free electrons per unit volume) for these
materials is very small as compared to metals, but
For a given conductor, l, A and n are constant, hence
increases very rapidly with rise in temperature (this
R  (1/)
happens due to breaking of covalent bonds). Though
If  represents the mean free path (Average distance  decreases but factor of n dominates. Therefore,
covered between two successive collisions) of the the resistance
electron and vrms, the root-mean-square speed, then
ml
R goes on decreasing with increase in
 V ne 2 A
 , Hence R  rms
v rms  temperature.
Now,
Note
(a)  decreases with rise in temperature because the
amplitude of vibrations of the +ve ions of the metal
increases and they create more hindrance in the
movement of electrons and,
Section C, D - Circuit theory, KCL & KVL,
(b) (i) vrms increases because vrms  T. Therefore, Battery, Grouping of cells
Resistance of the metallic wire increases with rise
in temperature. As  R and   (1/), hence 5. BATTERY AND EMF
resistivity increases and conductivity decreases with A battery is a device which maintains a potential
rise in temperature of the metallic of the metallic difference between its two terminals A and B.
wires. Figure shows a schematic diagram of a battery.
(ii) If R0 and Rt represent the resistances of metallic Some internal mechanism exerts forces on the
wire at 0°C and t°C respectively then Rt is given by charges of the battery material. This force drives
the following formula : the positive charges of the battery material towards
A and the negative charges of the battery material
Rt = R0(1 + t)
towards B. We show the force on a positive charge
where  is called as the Temperature coefficient 
q as Fb . As positive charge accumulates on A and
of resistance of the material of the wire.
negative charge on B, a potential difference develops
 depends on material and temperature but generally 
it is taken as a constant for a particular material for and grows between A and B. An electric field E is
small change. developed in the battery material from A to B and
 
Rt – R0 = R0  t exerts a force Fe  qE on a charge q. The direction

for very small change in temperature dR = R0  dt of this force is opposite to that of Fb In steady state,
the charge accumulation on A and B is such that
Fb  Fe . No further accumulation takes place.

: info@motion.ac.in, url : www.motion.ac.in, : 1800-212-1799


99, 8003899588
direction shown in the figure. The free electrons in
+
the wire move in the opposite direction and enter

Fb the battery at the terminal A. Some electrons are
A B
withdrawn from the terminal B which enter the wire
through the right end. Thus, the potential difference
If a charge q is taken from the terminal B to the
between A and B tends to decrease. If this potential
terminal A , the work done by the battery force Fb is
difference decreases, the electrostatic force F e
W = Fb d where d is the distance between A and B.
inside the battery also decreases. The force Fb due
The work done by the battery force per unit charge is
to the battery mechanism remains the same. Thus,
W Fbd there is a net force on the positive charges of the
E  battery material from B to A. The positive charges
q q
rush towards A and neutralise the effect of the
This quantity is called the emf of the battery. The electrons coming at A from the wire. Similarly, the
full form of emf is electromotive force. The name negative charges rush towards B. Thus, the potential
is misleading in the sense that emf is not a force, it difference between A and B is maintained.
is work done/charge. We shall continue to denote For calculation of current, motion of a positive
this quantity by the short name emf. If nothing is charge in one direction is equivalent to the motion
connected externally between A and B, of a negative charge in opposite direction. Using
Fb = Fe = qE or, Fbd = qEd = qV this fact, We can describe the above situation by a
where V = Ed is the potential difference between simpler model. The positive terminal of the battery
the terminals. Thus, supplies positive charges to the wire. These charges
are pushed through the wire by the electric field and
Fb d
E V they reach the negative terminal of the battery. The
q
battery mechanism drives these charges back to the
Thus, the emf of a battery equals the potential positive terminal against the electric fIeld existing in
difference between its terminals when the terminals the battery and the process continues. This maintains
are not connected externally. a steady current in the circuit
Potential difference and emf are two different Current can also be driven into a battery in the
quantities whose magnitudes may be equal in certain reverse direction. In such a case, positive charge
conditions. The emf is the work done per unit charge enters the battery at the positive terminal, moves
by the battery force Fb which is non-electrostatic in inside the battery to the· negative terminal and leaves
nature. The potential difference originates from the the battery from the negative terminal. Such a
electrostatic field created by the charges process is called charging of the battery. The more
accumulated on the terminals of the battery.
common process in which. the positive charge
A battery is often prepared by putting two rods or
comes out of the battery from the positive terminal
plates of different metals in a chemical solution.
is called discharging of the battery.
Such a battery, using chemical reactions to generate
emf, is often called a cell.
 EXAMPLE 8
E
i =2A
A B
vA R  2 vB
E = 10 V
A B
Find vA – vB
Now suppose the terminals of a battery are Sol. vA – iR – E = vB
connected by a conducting wire as shown in above vA – vB = iR + E = 4 + 10 = 14 volt
figure. As the terminal A is at a higher potential
than B, there is an electric field in the wire in the
EXAMPLE 9 0 – (–30)
Current in FC = = 15A from C to K
Shown in the figure. Find out the current in the wire BD 2
5
C B 0 – (–40)
current in DE =  20A from D to E
20v 10v 2
current in wire AH = 40 – 35 = 5 A from A to H
D A
2
6. KIR CHHO FF'S LAW S FO R
Sol. Let at point D potential = 0 and write the potential of
CIRCUIT ANALYSIS
10
other points then current in wire AD = = 5A from Before moving on to the statement of Kirchhoff's
2
law, we state some conventions to be followed in
20 circuit analysis :
A to D current in wire CB = = 4A from C to B
5 (1) Direction of conventional current is from high
potential to low potential terminal.
v=+20 5 v=0
C B (2) Current flows from high potential node A to low
4A
5A potential node B. if we traverse from point A to B,
4A
10v there is drop of potential; similarly from B to A,
there is gain of potential.
20v 1A + 10v
If we traverse from point A to B, there is drop of
potential; similarly from B to A, there is gain of
v=0 A
D 5A 2 potential. If a source of emf is traversed from negative
to positive terminal, the change in potential is +E.
 current in wire BD = 1 A from D to B
Potential gain
EXAMPLE 10 VA – VB = + E
Find the current in each wire High Potential Low potential
A B
Final Initial
H G F E Potential drop potential potential
10v 30v 40v
50v While discharging, current is drawn from the battery,
1 2 the current comes out from positive terminal and
2
A B C D enters negative terminal, while charging of battery
Sol. Let potential at point A is 0 volt then potential of current is forced from positive terminal of the battery
other points is shown in figure. to negative terminal. Irrespective of direction of
current through a battery the sign convention
40 – 0 mentioned above holds.
Current in BG = = 40 A from G to B
1 The positive plate of a capacitor is at high potential
+10V
and negative plate at low potential. If we traverse a
H G10V 35A F
E capacitor from positive plate to negative plate, the
10V 5A 10V 20A
30V 50V change in potential is –Q/C
40V
40V –30V –40V i + –
10V VB – VA = –Q/C
1 2 2 A B
5A Discharging Battery
40A 15A 20A
0V 0V i + – High Low
A B C 20V D
A B potential potential
0V 0V Charging Battery
If we traverse a resistor in the direction of current,
the change in potential is –IR. I1 I2
O
High Potential
+
Low Potential I3
– I6
A B VB – VA = –IR I4

Direction of traverse
Final Initial
Drop of potential
potential potential
The KCL applied to junction O yields.
Direction of traverse
I1 + I5 + I3 = I2 + I4
Gain of potential
Outgoing Incoming
If we traverse a resistor in the direction opposite current current
to the direction of current, the change in potential
is +IR.
2A
Potential gain

Potential drop VA – VB = +IR


5A
High Potential Low Potential Final Initial 3A
+ – potential potential
VA VB

Positive terminal of source of emf is at high potential EXAMPLE 11


and negative terminal at low potential. If we traverse
Find the potential at point A
a source of emf from the positive terminal to negative
terminal, the change in potential is –E. 20V

Direction of traverse 2
VB – VA =–E
VB – VA = – E 15V
1 2
Direction of traverse Final Initial –50 V
potential potential +10 V xA
VA – VB = + E 20V 5V

If a capacitor is traversed from negative plate to 1


positive plate, the change in potential is +Q/C.
–30V
VA – VB = + Q/C + –
Sol. Let potential at A = x, applying kirchhoff current
law at junction A

High Low x – 20 – 10 x – 15 – 20 x – 5  50 x  30
potential potential    0
A B 1 2 2 1

2x – 60  x – 35  x  45  2x  60
 0
(a) The Kirchhoff's Current Law 2
The Kirchhoff's current law (KCL) states that the  6x + 10 = 0
algebraic sum of the currents entering the junction
must equal to sum of the currents leaving the 5
 x–
junction. From the standard point of physics, KCL 3
is a statement of charge conservation.
–5
Potential at A = V
3
EXAMPLE 12 Sol. Assume x potential at the upper junction & zero
potential at the lower junction.
2 2 2 By KCl, we know that net current on a junction is
zero.
10V  i1 + i2 + i3 + i4 = 0
5V 20V
x–4 x–2 x2 x–4
   0
Find the current in every branch ? 2 4 4 2
Sol. Let we assume x potential at the top junction & 2x – 8 + x – 2 + x + 2 + 2x – 8 = 0
zero potential at lower junction
x i4 2
As from KCL,net current on a junction is O
i3
i1 + i2 + i3 = 0 i2
i1
4V 4
2V 4V
x – 5 x – 10 x – 20
  0 2
2 2 2 4
2V

35
3x = 35  x
3 O O O O

i1 x i3 8
6x – 16 = 0  x V
3
i2
2 1
2 2 2  i1  – A, i 2   A,
3 6

10V 20V
7 2
5V i3  A, i 4  – A
6 3
0 0 0
EXAMPLE 14
35
–5
10 2
 i1  3  A
2 3
2 2 2 2
5 25
Similarly, i2 = A ; i3 = – A.
6 6
4V 2V 2V

EXAMPLE 13

2
Find the current in every branch ?
Sol. The above question could be solved by assuming
potential x & y at the top junctions & zero potential
4V 4 at lower junctions
2V 4V
At the junction 1 applying KCL,
2
2V i1+ i2 + i3 = 0
4
x–4 x–2 x–y
  0
2 2 2

Find the current in every branch ?  3x – y = 6 ...(1)


anticlockwise
i1 x i3 2 i4 y i6 direction, starting from point A, we have
1
i 2 i5
2
2 2 2 2 + IR + Ir – E = 0
In the In the opposite From positive
4 2 2 opposite direction to to negative
4V 2V 2V direction current terminal
to current
O O O O
b c

At the junction 2 applying KCL,


i4 + i5 + i6 = 0 r
R
Real
y–x y–2 y Battery
  0
2 2 2 E
 3y – x = 2 ...(2)
a d
Solving (1) & (2)
9x – 3y = 18 E
Hence, I
 3y – x = 2 R r
 8x = 20
EXAMPLE 15
5 3 Find current in the circuit
x= ,y=
2 2
1
Just put the values of x & y & then the evaluate the
current in every branch 20 V 15 V
4
2
(b) The Kirchhoff's Voltage Law 10 V
The Kirchhoff's voltage law (KVL) states that the
algebraic sum of the potential difference around any 3 30 V
closed loop of an electric circuit is zero. The KVL
is a statement of conservation of energy. The Sol.  all the elements are connected in series
KVL reflects that electric force is conservative, the  current in all of them will be same
work done by a conservative force on a charge
20 V – i
taken around a closed path is zero. D +
C
We can move clockwise or anticlockwise, it will + 1 15 V
4i 4  –
make no difference because the overall sum of the – 2 2i
potential difference is zero. 10 V +
3 i
We can start from any point on the loop, we just A + – i B
have to finish at the same point. 3i 30 V
An ideal battery is modelled by an independent
let current = i
voltage source of emf E and an internal resistance
Applying kirchhoff voltage law in ABCDA loop
r as shown in figure A real battery always absorbs
10 + 4i – 20 + i + 15 + 2i – 30 + 3i = 0
power when there is a current through it, thereby
10 i = 25
offering resistance to flow of current.
 i = 2.5 A
Applying KVL around the single loop in
EXAMPLE 16 (a) Resistance in Series
Find the current in each wire applying only kirchhoff R1 R2 R3 R
voltage law
V1 V2 V3

10 50V
F E D
V V
2 2
(i) In this combination the resistance are joined end to
1
end. The second end of each resistance is joined to
30V first end of the next resistance and so on. A cell is
A B C connected between the first end of first resistance
and second end of last resistance. Figure shows three
Sol. Applying kirchhoff voltage law in loop ABEFA
resistances R1, R2 and R3 connected in this way.
i1 + 30 + 2 (i1 + i2) – 10 = 0
Let V1, V2 and V3 are the potential differences across
3i1 + 2i2 + 20 = 0 ...(i) these resistances.
Applying kirchhoff voltage law in BCDEB
(ii) In this combination current flowing through each
+ 30 + 2(i1 + i2) + 50 + 2i2 = 0 resistance will be same and will be equal to current
4i2 + 2i1 + 80 = 0 supplied by the battery.
2i2 + i1 + 40 = 0 ...(ii) (iii) As resistances are different and current flowing
Solving (i) and (ii) through them is same, hence potential differences
3[–40 –2i2] + 2i2 + 20 = 0 across them will be different. Applied potential
difference will be distributed among three resistances
–120 – 4i2 + 20 = 0
directly in their ratio.
10V 50V
As i is constant, hence V  R
F E D i.e., V1 = iR1, V2 = iR2, v3 = iR3
i1 i2 i2
i1 (iv) If the potential difference between the points A and
+
– 2(i1+i2) 2  – D is V, then
1 i1 – 2i2 2  V = V1 + V2 + V3 = i (R1 + R2 + R3)
+ 30V + (v) If the combination of resistances between two points
i1 i1+i2 i2 is replaced by a single resistance R such that there is
A B i2 C no change in the current of the circuit in the potential
difference between those two points, then the single
i2 = – 25 A resistance R will be equivlaent to combination and
V = i R i.e.,
and i1 = 10 A
iR = i (R1 + R2 + R3) or R = R1 + R2 + R3
 i1 + i2 = – 15 A (vi) Thus in series combination of resistances, important
current in wire AF = 10 A from A to E conclusion are
(a) Equivalent Resistance > highest individual
current in wire EB = 15 A from B to E
resistance
current in wire DE = 25 A from D to C (b) Current supplied by source = Current in each
resistance
7. COMBINATION OF RESISTANCE V V V V
or  1  2  3
A number of resistance can be connected in a circuit R1  R 2  R 3 R1 R 2 R 3
and any complicated combination can be, in general, (c) The total potential difference V between points
reduced essentially to two different types, namely A and B is shared among the three resistances
series and parallel combinations. directly in their ratio
V1 : V2 : V3 = R1 : R2 : R3
(b) Resistance in Parallel
V  1 1 1 
i1 R1 Thus,  V   
R R
 1 R 2 R 3
i A i2 R2 B
i R
i3 R3 1 1 1 1
or   
R R1 R 2 R 3

V (v) Thus in parallel combination of resistance important


V conclusion are :
(i) When two or more resistance are combined in such
(a) Equivalent resistance < lowest individual
a way that their first ends are connected to one
terminal of the battery while other ends are connected resistance
to other terminal, then they are said to be connected (b) Applied potential difference = Potential difference
in parallel. Figure shows three resistances R1, R2
and R3 joined in parallel between two points A and across each resistance.
B. Suppose the current flowing from the battery is i. or iR = i1R1 = i2R2 = i3R3
This current gets divided into three parts at the
(c) Current approaching the junction A = Current
junction A. Let the currents in three resistance R1 , R2
leaving the junction B and current is shared among
and R3, are i1, i2, i3 respectively.
the three resistances in the inverse ratio of resistances
(ii) Suppose potential difference between points A and
1 1 1
B is V. Because each resistance is connected i1 : i2 : i3 = : :
R1 R 2 R 3
between same two points A and B, hence potential
(i) If two or more resistance are joined in parallel
difference across each resistance will be same and
then i1R = i1R2 = i3R3............
will be equal to applied potential difference V. i.e., iR = constant i.e., a low resistance joined in
parallel always draws a higher current.
(iii) Since potential difference across each resistance
(ii) When two resistance R1 and R2 are joined in
is same, hence current approaching the junction A parallel, then
is divided among three resistances reciprocally in
i1 R 1 i12 R 12
their ratio. 1 or 1
i2 R 2 i 22 R 22
As V is constant, hence i  (1/R) i.e.,
i12 R1t R 2 H1 R 2
or  or 
V V V i2 2 R 2 t R1 H 2 R1
i1  ,i 2  and i 3 
R R2 R3 i.e., heat produced will be maximum in the lowest
resistance.
(iv) Because i the main current which is divided into
three parts i1, i2 and i3 at the junction A. EXAMPLE 17
Find current which is passing through battery.
 1 1 1 
hence, i  i1  i 2  i 3  V     2
 R1 R 2 R 3 
3
If the equivalent resistance between the points A
6
V
and B is R, then i =
R 1
30V
Sol. Here potential difference across each resistor is not 5V
30 V I= Modified circuit
3R
 battery has internal resistance here the concept R
of combination of resistors is useful.
Req = 1 + 1 = 2 P

30 R C R
i  15A A
2 B
R

EXAMPLE 18 5V
R
Find equivalent Resistance Current in P  3R  2V
1.5R  R 3R

× R R/2
A × B
R R R
P
A B

R R R
Sol. A VA
B
EXAMPLE 20
R VB VA VB
Find the current in 2  resistance.
Here all the Resistance are connected between the
4 2
terminals A and B. So, Modified circuit is
R 120V
So Req= 6 1
3 3
R
4 4
A R
B
Sol. 2, 1 in series = 3
R
18
3, 6 in parallel =  2
9
4
EXAMPLE 19 2, 4 in series = 6
12A 12A
Find the current in Resistance P if voltage supply 6, 3 is parallel = 2
between A and B is V volts Req = 4 + 4 + 2 = 10  2
R
120
i  12A
P R R 10 4 12A
A B
R 8
So current in 2 Resistance = A
3
3R
Sol. Req =
5 4 8
2 A A
12A 4A 3 3 3

3 6
A B
VA VB C 8A

4
• SPECIAL PROBLEMS B

R R
3 R
r F
r R R
1 6 R E G R
r A C
r r r
R R
A B
4 r H
R R
r r r R
2 7 D
r r
5
R

R
We wish to determine equivalent resistance between
R R R R
A and B. In figure shown points (1,2) (3, 4, 5) and
(6, 7) are at same potential Equivalent circuit can be R R E
A R R C R G
B,D F,H
redrawn as in figure shown.
The equivalent resistance of this series combination is R

r r r r 3r EXAMPLE 21
R eq.     
2 4 4 2 2
In the circuit shown in figure. (a) find the current
r 1,2 r 3,4,5 r 6,7 r flowing through the 100  resistor connecting points
r r U and S.
A B
r r r r
Sol. Figure (b) shows simplified circuit. The battery is
r r
directly attached to resistor 90hence current in it
In the figure shown, the resistances specified are is 2 A, see figure (c), The total resistance of second
in ohms. We wish to determine the equivalent branch is also 90 , hence current divides equally.
resistance between point A and D. Point B and C,
Now current through 45  resistor is 2 A and it is a
E and F are the the same potential so the circuit
can be redrawn as in figure shown. combination of two equal 90  resistors. Once again
Thus the equivalent resistance is 1 . current divides equally. 90  resistor is a series
combination of 40  and 50 , hence current
A
through them is equal, i.e.,
2
D 2
1 1
1 1 1 2 1

E 1 F A 1 2 1
2 2 B,C E,F D
B C
1

In the network shown in figure shown all the


resistances are equal, we wish to determine equivalent
resistance between A and E. Point B and D have
same potential, similarly F and H have same potential.
The equivalent circuit is shown in figure shown. The
equivalent resistance of network is 7R/2.
the junction B gets further divided into two parts
(i 1 – i g) and i g, along branches BC and BD
respectively. At junction D, currents i2 and ig are added
to give a current (i2 + ig), along branch DC. (i2 – ig)
and (i2 + ig) add up at junction C to give a current (i1
+ i2) or i along branch CE. In this way, currents are
distributed in the different branches of bridge. In this
position, we get a deflection in the galvanometer.
(iv) Now the resistance P,Q,R and S are so adjusted
that on pressing the key K2, deflection in the
galvanometer becomes zero or current ig in the
branch BD becomes zero. In this situation, the
bridge is said to be balanced.
1 A. As 50  resistor is a parallel combination of (v) In this balanced position of bridge, same current i1 flows
two equal 100  resistors, they must have the same in arms AB and BC and similarly same current i2 in
current i.e., 0.5 A arms AD and DC. In other words, resistances P and Q
and similarly R and S, will now be joined in series.
8. WHEATSTONE'S BRIDGE (vi) Condition of balance : Applying Kirchhoff's 2nd
law to mesh ABDA, i1P + igG – i2R = 0 ....(1)
B
i1 – ig Similarly, for the closed mesh BCDB, we get,
P ig Q (i1 – ig) Q – (i2 + ig)S – igG = 0 ....(2)
i1 When bridge is balanced, ig = 0. Hence eq. (1) &
A C
G (2) reduce to
i2
i2+ig i1P – i2R = 0 or i1P = i2R ....(3)
K2
R S
i1Q – i2S = 0 or i1Q = i2S ....(4)
i1+i2 D P R
i1+i2 
Dividing (3) by (4), we have, ....(5)
Q S
+ –
This is called as condition of balanced for
E K1
Wheatstone's Bridge.
(i) Wheatstone designed a network of four resistances (vii) It is clear from above equation that if ratio of the
with the help of which the resistance of a given resistance P and Q, and the resistance R are known,
conductor can be measured. Such a network of then unknwon resistance S can be determined. This
resistances is known as Wheastone's bridge. is the reason that arms P and Q are called as ratio
(ii) In this bridge, four resistance P, Q, R and S are so arms, arm AD as known arm and arm CD as
connected so as to form a quadrilateral ABCD. A unknown arm.
sensitive galvanometer and key K2 are connected (viii) When the bridge is balanced then on inter-changing
between diagonally opposite corners B and D, and the positions of the galvanometer and the cell there is
a cell and key K1 are connected between two other no effect on the balance of the bridge. Hence the arms
corners A and C (figure shown) BD and AC are called as conjugate arms of the bridge.
(iii) When key K1 is pressed, a current i flows from the (ix) The sensitivity of the bridge depends upon the value
cell. On reaching the junction A, the current i gets of the resistances. The sensitivity of bridge is
divided into two parts i1 and i2. Current i1 flows in the maximum when all the four resistances are of
arm AB while i2 in arm AD. Current i1, on reaching the same order.
EXAMPLE 22 8.1 Unbalanced Wheatstone Bridge
Find equivalent resistance of the circuit between the EXAMPLE 24
terminals A and B.
10 B(x) 5
A C
10 6 i1 i5 i3
i i
v 2 0
A 20 B i2 i4
D F
5 E(y) 10

5 3
v 0
Sol. Since the given circuit is wheat stone bridge and it v
is in balance condition.
Find equivalent resistance ?
 10 × 3 = 30 = 6× 5
Sol. Let potential at point B is x and E is Y
hence this is equivalent to
v
16  8 16 R eq 
R eq    i
16  8 3
Applying KCl at point B
10 6 x–v x–y x–0
  0
10 2 5
A B
8x – 5y = v ...(1)
5 Applying KCL at point E
3
y–v y–x y–0
  0
5 2 10
EXAMPLE 23
 8y – 5x = 2v ...(2)
Find the equivalent resistance between A and B
R 6v 7v
solving x & y x , y
13 13
R R
A B current from branches BC & EF adds up to give
R total current (i) flowing in the circuit.
R
x – 0 y – 0 19v
i = i3 + i4 =  
Sol. This arrangement can be modified as shown in 5 10 130
figure since it is balanced wheat stone bridge.
V 130
2R  2R  i  Req. =
R eq  R R eq 19
2R  2R

C
R R Ladder Problem :
R R R R R
R A 
A B
R R R R R
R R
B 
D
Find the effective resistance between A & B ?
Sol. Let the effective resistance between A & B be RE between points C & D would be kRE (where RE is
since the network is infinite long, removal of one the effective resistance)
cell from the chain will not change the network.  Effective R between A & B.
The effective resistance between points C & D
would also be RE. R(kR E )
RE  R 
The equivalent network will be as shown below kR E  R
On solving we get
R C
A 2kR – R  (R – 2kR)2  4kR 2
RE 
2k
R RE
15. Symmetrical Circuits :
B Some circuits can be modified to have simpler
D
solution by using symmetry if they are solved by
traditional method of KVL and KCL then it would
The original infinite chain is equivalent to R in series
take much time.
with R & RE in parallel.
EXAMPLE 26
RR E Find the equivalent Resistance between A and B
RE  R 
R  RE
C
RER + RE2 = R2 + 2RRE R R
R
2 2
 R – RRE – R = 0
E R R
A B

R(1  5) R
RE  R R
2 D

EXAMPLE 25 Sol. I Method : MIRROR SYMMETRY


The branches AC and AD are symmetrical
R kR k2R k3R  current through them will be same.
A
The circuit is also similar from left side and right
R kR k2R k3R side like mirror images with a mirror placed alone
CD therefore current distribution while entering
B
through B and an exiting from A will be same. Using
Find the equivalent resistance between A & B ? all these facts the currents are as shown in the figure.
It is clear that current in resistor between C and E is
Sol. As moving rom one section to next one, resistance
0 and also in ED is 0. It's equivalent is shown in
is increasing
figure (b)
R C C
A
R R
R
R kRE i i1 i–2i1 i1
A B
i–2i1 E i1 i
B
D i1 R R i
R
by k times. Since the network is infinitely long, D
removal of one section from the chain will bring a
(fig.a)
little change in the network. The effective resistance
branches AC and AD at the input
R R  current in them are same but from input and
R R from exit the circuit is not similar
A B ( on left R and on right 2R)
R R  on both sides the distribution of current will not
be similar.
(fig.b)
Here Vc = Vd

C
2R
R eq 
3 R 2R
II Method : FOLDING SYMMETRY R
x y
R 2R
 The potential difference in R between (B, C) A B
i i–2x E
and between (B, D) is same VC = VD y
R R 2R
Hence the point C and D are same hence circuit x
can be simplified as
R E R D
A B hence C and D are same point
R R/2
R R
So, the circuit can be simplified as
Now it is balanced wheat stone bridge.
R R
C,D R E 2R
A B
This called folding.
Now, it is Balanced Wheatstone bridge R/2
R/2 R
R E R
A B C,D
R/2 R/2
R/2 3R 9
3R  R
R eq  2  2 R
(C,D) 3R 9
3R 
2 2
2R  R 2R
R eq  
2R  R 3
EXAMPLE 28
In II Method it is not necessary to know the currents
Find the equivalent Resistance between A and B
in CA and DA
R
R R R
EXAMPLE 27 R
R R
Find the equivalent Resistance between A and B A B
R R
C R R
R 2R
R R
R 2R A R
A B B
E R
R R R
R R
2R x R R x
Sol. A x i–2x B
D i R i–2x x
R R R
Sol. In this case the circuit has symmetry in the two
C R D
Here VA = VC and VB = VD
Here the circuit can be simplified as
y R R y
A B 1 6
i i–y i–y 2,4,8 3,5,7
R/2
R/2 R/2
R/2
R/2
R/3 R/6 R/3.
4R
2R 
R eq 3  4R
10R 5 Ans.
5R
3 R eq 
6
R R
A B (b) When current enters at 1 and leaves at 2
R/2 R/2
R/2 R/2 Sol. Here 3, 7 are equipotential surface (if we move
from 1  3, 7 we have along face and 2, 3, 7 we
R/2
move along edge) similarly 4, 8 are equipotential
2R
A B
surface.

R/2 R/2 1 2
R/3
2R
A B

4R/3 3, 7 4,8

EXAMPLE 29
Twelve equal resistors each R  are connected to 5 6
form the edges of a cube. Find the equivalent
resistances of the network. 7R
R eq 
5 6 12
(c) When current enters at 1 and leaves at 3
8 7 Sol. If we cut the cube along the plane passing through
2, 4, 5, 7 then by mirror symmetry, the final
configuration will be
4 3
5 6 5 6
2
1
(a) When current enters at 1 & leaves at 6 (body 8 7 8 7

diagonal)
Sol. Here 2, 4, 8 are equipotential points (if we move 4 3 4 3

from 1  2, 4, 8 it comes along the edge & 6  2, 2 2


1 1
4, 8 it comes along face diagonal). Similarly 3, 5, 7
are equipotential points.
3R
R eq 
4
9. COMBINATIONS OF CELLS (B) Cells in Parallel
A cell is used to maintain current in an electric
circuit. We cannot obtain a strong current from a E
1 r
single cell. Hence need arises to combine two or
more cells to obtain a strong current. Cells can be E E r/n
2 r
combined in three possible ways :

......
I
(A) In series, (B) In parallel, and E
n r
(C) In mixed grouping. R
(A) Cells in Series R

1 2 n nE
r r ...... r nr
(I) When E.M.F's and internal resistance of all
E E E I the cells are equal : In this combination, positive
I
terminals of all the cells are connected at one point
R
R and negative terminals at other point. Figure shown
such cells connected in parallel across some external
In this combination, cells are so connected that –ve resistance R. Let e.m.f and internal resistance of
terminal of each cell is connected with the +ve each cell are E and r respectively.
terminal of next and so on. Suppose n cells are Because all the cells are connected in parallel
connected in this way. Let e.m.f and internal between two points, hence e.m.f of battery = E.
resistance of each cell are E and r respectively. Total internal resistance of the combination of n cells = r/n
Net e.m.f of the cells = nE. Total internal resistance Because external resistance R is connected in
= nr. Hence total resistance of the circuit = nr + R. series with internal resistance, hence total resistance
If total current in the circuit is I, then of the circuit = (r/n) + R
If current in external resistance is I, then
net e.m.f nE
I  ...(1)
Total Resistance nr  R net E.M.F E nE
I  
Total resistance (r / n)  R r  nR
Case (i) : If nr < < R, then I  n E / R i.e., if total internal
resistance of the cells is far less than external nE E
Case (I) : If r << R, the I   i.e., if internal
resistance, then current obtained from the cells is nR R
approximately equal to n times the current obtained resistance of the cells is much less than external
from a single cell. Hence cells, whose total internal resistance, then total current obtained from
resistance is less than external resistance, just be combination is nearly equal to current given by one
joined in series to obtain strong current. cells only. Hence there is no use of joining cells of
low internal resistance in parallel.
nE E
Case (ii) : If nr >> R, then I   i.e., if total internal nE
nr r Case (II) : If r >> R, then I  i.e., if the internal
r
resistance of the cells is much greater than the
resistance of the cells is much higher than the external
external resistance, then current obtained from the
resistance, then total current is nearly equal to n
combination of n cells is nearly the same as obtained
times the current given by one cell. Hence cells of
from a single cell. Hence there is no use of joining high internal resistance must be joined in parallel to
such cells in series. get a strong current.
(II) When emf's and internal resistance of all the cells If n cells are joined in parallel, then
connected in parallel are different : In this case,
Ei Ei
total current in external resistance is obtained with r r 1
i i
the help of Kirchhoff's laws. Figure shows three I i and Eeq.  , req. 
1 1 1
cells of e.m.f E1, E2 and E3 and internal resistances 1  R r r
i ri i i
r1, r2 and r3 connected in parallel across some
external resistance R. Suppose currents given by (C) Cells in Mixed Grouping
three cells are i1, i 2 and i 3. Hence according to
(1) (1) (2) (n)
Kirchhoff's first law, total current I in external .......
resistance R, is given by (2)
.......
I = i1 + i2 + i3 ...(1)

.....

.....
Applying Kirchhoff's 2nd law to closed mesh ABEF .......
we get (m)

 (E1 – IR)  R
IR + i1r1 = E1 or i1 =  r1  ...(2)
  In this combination, a certain number of cells are
Similarly, for closed meshes ABDG and ABCH, we joined in series in various rows, and all such rows
are then connected in parallel with each other.
get
Suppose n cells, each of e.m.f E and internal
E 2 – IR resistance r, are connected in series in every row
i2  ....(3)
r2 and m such rows are connected in parallel across
some external resistance R, as shown in figure.
E1 r1
i1 Total number of cells in the combination = mn. As e.m.f.
F E of each row = nE and all the rows are connected in parallel,
i2 E2 r2 hence net e.m.f of battery = nE.
G D
Internal resistance of each row = nr. As m such
i3 E3 r3 rows are connected in parallel, hence total internal
H C
I  nr 
R resistance of battery =  
m
A B

 nr  
E – IR Hence total resistance of the circuit =    R 
and i3  2 ....(4)  m  
r3
Substituting eq. (2), (3) and (4) in eq. (1), we have If the current in external resistance is I, then
E1 – I R E 2 – I R E 3 – I R
I   net e.m.f nE mnE
r1 r2 r3 I  
Total resistance (nr / m)  R nr  mR
E1 E 2 E 3 1 1 1
   –IR   
r1 r2 r3  r1 r2 r3  mnE
 2

  1 1 1  E E
 nr – mr   2 nmrR
E
or I 1  R      1  2  3
  r1 r2 r3  r1 r2 r3 It is clear from above equation that I will be maximum
when
(E1 / r1 )  (E 2 / r2 )  (E 2 / r3 )
or I [(nr – mR)2 + 2  nmrR] is minimum.
1  R(1/ r1  1/ r2  1/ r3 )
This will be possible when the quantity [nr – mR]2 10V 2
is minimum. Because this quantity is in square, it can 6V 1
not be negative, hence its minimum value will be equal B
Sol.
to zero, i.e., A
C 2
4V
nr
mR = nr or R= Battery (B) and (C) are in parallel combination with
m
opposite polarity. So, their equivalent
i.e., In mixed grouping of cells, current in external
10 –4
resistance will be maximum when total internal 
BC  2 2  5 – 2  3V
resistance of battery is equal to external resistance. 1 1 1

Because power consumed in the external resistance 2 2
or load = I2R, hence when current in load is rBC = 1 
maximum, consumed power in it is also maximum, Now, ABC  6 – 3  3V
Hence consumed power in the load will also be 3V
6V 1 1
nr
maximum when R = . rABC = 2 Ans.
m

mnE mnE nE mE Note


I max  or  or
2mR 2nr 2R 2r

EXAMPLE 30
Find the current in the loop.
Section E - Electrical Power & Energy
1 40v, 2 
10. ELECTRICAL POWER
4 2 The energy liberated per second in a device is called
its power, the electrical power P delivered by an
15v, 1 10v electrical device is given by
1
3 20v,1 dq
P= V = VI
dt
Sol. The given circuit can be simplified as
35 v, 5 Power consumed by a resistor.
10
V2
P = VI = I2R = watt
R
The power P is in watts when I is in amperes, R is
35 35 7 7
i   A  I A in ohms and V is in volts.
10  5 15 3 3
The practical unit of power is 1 kW = 1000 W.

EXAMPLE 31 V2
The formula for power P = I2R = VI = is true
Find the emf and internal resistance of a single R
battery which is equivalent to a combination of three only when all the electrical power is dissipated as
batteries as shown in figure.
heat and not converted into mechanical work, etc.
10V 2 simultaneously.
6V 1 If the current enters the higher potential point of the
device then electric power is consumed by it (i.e. acts
as load). If the current enters the lower potential point
4V 2
then the device supplies power (i.e. acts as source.)
(A). JOULE'S LAW OF ELECTRICAL HEATING
When an electric current flows through a conductor 50W, 220V
100W, 220V
20W, 220V
electrical energy is used in overcoming the resistance
of the wire. If the potential difference across a
B1 B2 B3
conductor of resistance R is V volt and if a current
of I ampere flows the energy expanded in time t
seconds is given by
V2
W = VIt joule = I2Rt joule = t 220V
R
The electrical energy so expanded is converted into 2
Vrated
heat energy and this conversion is called the heating Sol. Prated 
R
effect of electric current.
The heat generated in joules when a current of I 2
Vrated
amperes flows through a resistance of R ohm for t R
Prated
seconds is given by
I2 Rt  R 3  R 2  R1
H = I2Rt joule = cal.
4.2 Power = i2R
This relation is known as Joule's law of electrical As current passing through every bulb is same
heating.
 Brightness order is B3 > B2 > B1
EXAMPLE 32
EXAMPLE 34
If bulb rating is 100 watt and 220 V then determine
(a) Resistance of filament The above B B 1 2

(b) Current through filament c o n f i gu r a t i o n


(c) If bulb operate at 110 volt power supply then shows three B 3

find power consume by bulb. identical bulbs,


Sol. Bulb rating in 100 W and 220 V bulb means when Grade them in
220 V potential difference is applied between the order of their
two ends then the power consume is 100 W brightness. V
Here V = 220
Sol. B1 & B2 withdraw less current as compared to B3
P = 100
because in series they give 2R resistance where as
V2 R is the resistance dut to B3.
 100
R Power = i2R
So R = 484   Brightness order : B3 > B2 = B1.
Since Resistance depends only on material hence it
is constant for bulb
EXAMPLE 35
V 220 5
I   Amp. B6
R 22  22 11
Grade the bulbs
power consumed at 110 V
in order of their B3
B5 B4
110  110 brightness
 power consumed =  25W
484 (All bubls B1 B2
are identical)
EXAMPLE 33
In the following figure, grade the bulb in order of
their brightness : V
For a given real battery the load resistance maximizes
1 B6
Sol. As i the power if it is equal to the internal resistance of the
R
battery.
3 1 B5 B3 P
i1 : i 2  : =6:5 i4 B4 2 R
5R 2R  E 
P(R)    R
R r 
6i 5i i1 i3 R R
 i1  r
, i2  B1 B2
11 11
E
1 1 i2 O r
i3 : i 4  : =1:2 i
2R R The maximum power transfer theorem in general,
As i3 + i4 = i1 holds for any real voltage source. The resitance R
V
may be a single resistor or R may be the equivalent
2i 4i
 i3  ; i4  resistance of a collection of resistors.
11 11
power = i2R Note
 Order of Brightness : B5 > B1 = B2 > B6 >
B4 = B3

(B) MAX IMUM POW ER T RANS FER Section F - Electrical Instrument + Exp.
Verifying ohm's law & Specific
THEOREM Resistance Using Meter Brige &
Let E be emf and r internal resistance of the battery. Post Office + Potentiometer
It is supplying current to an external resistance R (EMF & Int. Res.)

E 11. INTRUMENTS
current in circuit I = (A) AMMETER
(R  r )
It is a device used to measure current and its always
connected in series with the 'element' through which
The power absorbed by load resistor R is current is to be meaured, e.g., in figure (A) ammeter
2 A1 will measure the current (I1) through resistance
 E  R1, A2 measures current (I2) through R2 and R3 while
P = I2R =   R
R r  A, measures current I( I1 + I2).
Regarding an ammeter it is worth noting that :
For maximum power transfer we take the derivative of P
(1) The reading of an ammeter is always lesser than actual
w.r.t R, set it equal to zero and solve the equation for R. current in the circuit, e.g., true current in the resistance
dP V
0 R in the circuit shown in figure (B) is I =
dR R
However, when an ammeter of resistance r is used to
dP (R  r)2 – R[2(R  r)]
 E2 0 measure current as shown in figure (C), the reading will be
dR (R  r) 2
V  V
R I'   I 
(R  r)  R
I
– A +
E I
r battery R1 I1
+A –
1
Solving for R, we have
(R + r)2 – R (2) (R + r) = 0 I2
(R + r) – 2R = 0 R2 I2 R3
R=r A2
+ –
(A)
EXAMPLE 37
V V Find the current in the circuit also determine

percentage error in measuring in current through an
I I I' I' A r ammeter (a) and (b).
+
2 2

R R A 0.5 
10V 10V
(B) (C)
(a) (b)
(2) Smaller the resistance of an ammeter more accurate 10 10
will be its reading. An ammeter is said to be ideal Sol. In A I  5A , In B I  4A
2 2.5
if its resistance (r) is zero. However, as practically
r  0, ideal ammeter cannot be realised in practice. i – i'
Percentage error is =  100 = 20% Ans.
(3) To convert a galvanometer into an ammeter of a i
certain range say I, a small resistance S (called Here we see that due to ammeter the current has
shunt) is connected in parallel with the galvanometer reduced. A good ammeter has very low resistance
so that the current passing through the as compared with other resistors, so that due to its
galvanometer of resistance G becomes equal to its
presence in the circuit the current is not affected.
full scale deflection value Ig. This is possible only if
IgG = (I – Ig)S
EXAMPLE 38
(i – iG ) S 3
Find the reading of A
i ig
G ammeter. Is this the
external
resistance current through 6  ? 6
18V 1

Ammeter 3 6
Sol. R eq   1  3
3 6
Ig Current through battery
i.e., S = (I – I ) G
g
18
I  6A
3
EXAMPLE 36 So, current through ammeter
What is the value of shunt which passes 10% of the 6
 6
 4A
main current through a galvanometer of 99 ohm ? 9
Sol. As in figure RgIg = (I – Ig)S No, it is not the current through the 6 resistor.
Ideal ammeter is equivalent to zero resistance
I  I 
 99    I –  S wire for calculation potential difference across
10  10 
Rg it is zero.
 S = 11  I Ig
G
For calculation it is (B) VOLTMETER
simply a resistance It is a device used to measure potential difference and is
(I–Ig) S

Resistance of ammeter I
I
A R1 R2 I' +
R .S V v r
RA  G R R

RG  S V1 V2
+ – + – I
for S << RG  RA = S V –
+
(B)
(A)
always put in parallel with the 'circuit element' across 200  200 
which potential difference is to be measured e.g., in
Figure (A) voltmeter V1 will measure potential 100 V 300  100 V 300  v 600 
difference across resistance R1, V2 across resistance
R2 and V across (R1 + R2) with V = V1 + V2 (A) (B)

Regarding a voltmeter it is worth noting that :


100
(1) The reading of a voltmeter is always lesser than true Sol. In (A) : Potential difference =  300
200  300
value, e.g., if a current I is passing through a resistance
R [Fig. (B)], the true value V = IR. However, when a = 60 volt
voltmeter having resistance r is connected across R, In (B) Potential difference
the current through R will become 100  300  600 
=   50 volt
r V 300  600  300  600 
I'  I and so V'  I ' R  200 
(R  r) [1  (R / r)] 300  600
and as voltmeter is connected across R its reading We see that by connected voltmeter the voltage
V' is lesser than V. which was to be measured has charged. Such
(2) Greater the resistance of voltmeter, more accurate voltmeters are not good. If its resistance had been
will be its reading. A voltmeter is said to be ideal if very large than 300  then it would have not
its resistance r is infinite, i.e., it draws no current affected the voltage by much amount.
from the circuit element for its operation. Ideal
voltmeter has been realised in practice in the form (C) METRE-BRIDGE
of potentiometer. Metre-bridge is a sensitive device based on the
(3) To convert a galvanometer into a voltmeter of certain principle of wheatstone-bridge, for the determination
range say V, a high resistance R is connected in of the resistance of a conductor (wire). Its sensitivity
series with the galvanometer so that current passing is much more than that of the post-office box.
through the galvanometer of resistance G becomes
equal to its full scale deflection value Ig. This is
possible only if

v
Ig

G
R

V
V = Ig (G + R) i.e., R –G
Ig Metre-bridge is shown in figure AC is one metre long
wire of manganin or constantan which is fixed along a
EXAMPLE 39 scale on a wooden base. The area of cross-section of
A voltmeter has a resistance of G ohm and range of the wire is same at all places. The ends A and C of the
V volt. Calculate the resistance to be used in series wire are joined to two L-shaped copper strips carrying
with it to extend its range to nV volt. binding-screws as shown. In between these strips,
V leaving a gap on either side, there is a third copper
Sol. Full scale current ig = strip having three binding screws. The middle screw
G
D is connected to a sliding jockey B through a shunted
to change its range
- galvanometer G. The knob of the jockey can be made
V1 = (G +Rs) ig
to touch at any point on the wire.
V To measure the unknown resistance, the connection
 nV = (G + Rs)
G as shown in figure are made.
Rs = G(n – 1) Ans. A resistance R is taken out from the resistance box
and the key K is closed. Now the jockey is slided
EXAMPLE 40 along the wire and a point is determined such that,
Find potential difference across the resistance 300 on pressing the jockey on the wire at that point there
 in A and B.
is no deflection in the galvanometer G. In this (D) POTENTIOMETER
position the points B and D are at the same potential. A potentiometer is used to compare e.m.fs. of two
The point B is called 'null-point'. The lengths of both cells or to measure internal resistance of a cell.
the parts AB and BC of the wire are read on the Principle : The potentiometer is based upon the
scale. Suppose the resistance of the length AB of principle that when a constant current is passed
the wire is P and that of the length BC is Q.
through a wire of uniform area of cross-section, the
Then, by the principle of Wheastone-bridge. We have,
potential drop across any portion of the wire is
P R directly proportional to the length of that portion.

Q S Construction : A potentiometer consists of a number
of segments of wire of uniform area of cross section
Let the length AB be l cm. Then the length BC will stretched on a wooden board between two thick copper
be = (100 – l) cm. strips. Each segment of wire is 100 cm long. The wire
is usually of constantan or manganin. A metre rod is
l
 resistance of AB, i.e. P = , and resistance of fixed parallel to its length. A battery connected across
A the two end terminals sends current through the wire,
BC, Q =  (100 – l)/A which is kept constant by using a rheostat.
where  is the specific resistance of the material of Theory : Let V be potential difference across certain
the wire and 'A' is the area of cross-section of the portion of wire, whose resistance is R. If I is the
wire. Thus current through the wire, then V = IR

P l
 ...(i)
Q (100 – l )

P
Substituting this value of in eq. (i), we get
Q

l R R(100 – l )
 or S
(100 – l ) S l
R is the resistance taken in the resistance box and l is
the length measured. Hence, the value of resistance l
We know that R =  ,
S can be determined from the above formula. A
where l, A and  are length, area of cross-section
A number of observations are taken for different
and resistivity of the material of wire respectively.
resistances in the resistance box and for each
observation the value of S is calculated. l
 V  I
Finally, the experiment is repeated by interchanging A
the unknown resistances S and the resistance box. If a constant current is passed through the wire of uniform
The mean of the values of S is then obtained. area of cross-section, then I and A are constants. Since,
for a given wire,  is also constant, we have
V = constant × l or Vl
EXAMPLE 41 Hence, if a constant current flows through a wire of
In a meter bridge experiment, the value of unknown uniform area of cross-section, then potential drop along
resistance is 2. To get the balancing point at 40 cm the wire is directly proportional to the length of the wire.
distance from the same end, then what will be the Applications of a potentiometer. A potentiometer
resistance in the resistance box ? can be put to following uses :
Sol. Apply condition for balance wheat stone bridge,
1. To compare e.m.fs. of two cells : Two cells, whose
P  P 100 – 40 e.m.fs. are E1 and E2, can be compared by making
  
Q 100 –  2 40 use of the ciruit as shown in figure. The positive poles
of both the cells are connected to the terminal A of
P=3 Ans.
the potentiometer. The negative poles of the two cells
are connected to terminals 1 and 2 of a two way key.
while its common terminal is connected to a jockey j
through a galvanometer G. An auxiliary or driver Rh
– + + E' –
battery of e.m.f E, an ammeter A, rheostat Rh and a A K1
one way key K are connected between the end
terminals A and B of the potentiometer. Thus, the
B
positive poles of the two cells as well as the positive
J
pole of auxiliary battery are connected at the common
point A. It may be pointed that the e.m.f of auxiliary A
battery should always be greater than the e.m.f of
either of the two cells. E
G R
K + –

– + + E' –
A S
Rh K2
A constant current I is maintained through the
B
200 300 potentiometer wire with the help of the rheostat.
Plug in the key K2 is kept out and the jockey is
200
moved over the potentiometer wire so as to balance
100 the e.m.f. E of the cell, whose internal resistance is
A to be found. Let l1 be the balancing length of the
E1
potentiometer wire between point A and jockey J. If
1
+ – x is resistance per unit length of the wire, then
+ –
G E = (x l1) I
3 With the help of resistance box S, introduce
E2
2 resistance say S and then put the plug in key K2.
To compare the e.m.fs of the two cells, a constant Now find the balance point for the terminal potential
current is passed through the potentiometer wire difference V between the two poles of the cell. If l2
between points A and B. The current is kept constant is the balancing length, then
by using the rheostat. V = (x l2) I
When the plug is put in the gap between the terminals Dividing above equation , we have
1 and 3 of the two way key, the cell of e.m.f. E1
E1 l1
will come in the ciruit. Suppose the balancing length 
(between points A and J) is l1. If x is the resistance E 2 l2
per unit length of the potentiometer wire and I, the The internal resistance* of the cell is given by
constant current flowing through it, then
E 
E1 = (xl1) I r   – 1 S
When the key is put in the gap between the terminals V 
2 and 3 and removed from the gap between 1 and Using above equation , we have
3, the cell of e.m.f E2 wil be included in the circuit.
l  l –l
Let the balancing length be l2 in this case. Then, r   1 – 1 S or r  1 2 S
E2 = (x l2) I l
 2  l2
Dividing above equation Knowing the values of l1, l2 and S, the internal
E1 l1 resistance r of the cell can be found.

E 2 l2 Note

Note

We use potentiometer for two tasks :


(i) to find emf of a cell
2. To measure internal resistance of a cell. The internal (ii) to find internal resistance of a cell
resistance of a cell may be found by using a We will first analyse the first task  to find emf of
potentiometer by setting up the circuit as shown in figure. a cell through some examples
EXAMPLE 42 150V
Potential gradient = 10 m  15V / m

200V
25 
50V 150 O
–50

10m
A 75 
10m
x P
150 O
G x
r
G
20V

Find the value of x if A is the null point ? 120V 30

For 120 V
50
O
120V / m
50V   8m
15V
 x = 10 – 8 = 2m
Sol.
10m Now we will analyse the other task to find internal
50 O resistance of the cell using potentiometer. The main
key point is that first analyse the main circuit then
30 G
the auxillary circuit (supplementary circuit)
20V E1
Potential gradient =

50
Potential gradient = = 5 V/m Now for the auxillary circuit
10
E1
 For 20V potential difference
O E1
20
  4m
5
 x = 10 – 4 = 6m

x P
EXAMPLE 43 O E1

25  G null deflection
E
r
ER
200V E R r

75  R
10m
P E
i
x Rr
2 G Er ER
E ir  E  
Rr R r
120V
ER E
Find the value of x if P is a null point.   1x
Rr 
Sol. Current in the main circuit
Let we take some examples to understand the topic
200 – 25 i – 75 i = 0
i=2A in better way.
EXAMPLE 44 Sol. 100 – 5i1 – 20 i1 = 0
90V 10 i1 = 4 A

80
10cm
Potential gradient = = 8V/m
20 10
P
x 8 – 2i2 – 2i2 – 2 = 0
G
2 100V
0 80
20V 2
100 5

Find x if P is a null point?


10m
Sol. First analysing the main circuit,
90 – 10i1 – 20 i1 = 0 0 80
P
i1 = 3 A. G
60V 20
Potential gradient = = 6V/m. 2V 2 2
10m 0 5
90V 10
0 60 8
90 0
8V 2 5
10cm 20
3
0
60 i2 = A
G
2
20 2 10
0 5v 5
for 5 volts   m
20V 2 8v / m 8

Now analysing the auxillary circuit 5


20 – 2i2 – 2i2 = 0  x= m
8
i2 = 5 A.
10 5
For 10 Ve   m
6 3 (E) POST OFFICE BOX
5 25 A post office Box can also be used to measure an
 x = 10 –  m
3 3 unknown resistance. It is a wheatstone Bridge with
EXAMPLE 45 three arms P, Q and R: while the fourth arm(s) is
the unknown resistance. P and Q are known as
100V the ratio arms while R is known as the rheostat
5 arm.
At Balance. the unknown resistance
10m 20 S = (P/Q) R ....(1)
0 P 80V The ratio arms are first adjusted so that they carry
x
G 100 W each. The resistance in the rheostat arm is
2V 2
now adjusted so that the galvanometer deflection
is in one direction. If R = RO (ohm) and
R = RO +1 (ohm) are the resistance in rheostat
2 arm. for which the deflection in galvanometer is in
8V
opposite direction, then it implies that the unknown
Find x if P is a null point ? resistance’s lies between RO & (RO+1) ohm.
X Now, the resistance in P and Q are made 100 W
R1 B R2 C and 1000 W respectively and the process is re-
A
peated. Equation (1) is used to compute S. The ra-
1000 100 10 1C 100 1000
tio P/Q is progressively made 1:10 and then 1:100.
R
Thus, the resistance S can be accurately measured.
1NF Errors
1 2 2 5 10 20 20 50

5000 2000 2000 1000 500 200 200 100 The major sources of error are the connecting wires
unclear resistance plugs. change in resistance due
A' K1 B' to joule heating, and the insensitivity of the
K2
wheatstone bridge.
G
(A)
These errors may be removed by using thick
connecting wires, clean plugs, keeping the circuit
B
on for very brief periods (to avoid joule heating and
R1 R2 calculating the sensitivity.)
G In order that the sensitivity is maximum, the
A C resistance in the arm P is kept close to the value of
K2
the resistance S.
R X
Note
D

E K1
(B)
Exercise - 1 Objective Problems | JEE Main
Section A - Definition of Current, Current 7. A wire has a non-uniform cross-section as shown in
Density, Drift Velocity figure. A steady current flows through it. The drift
1. The electric current in a discharge tube containing a speed of electrons at points P and q is vp and vQ.
gas is due to
(A) electron only
(B) positive ions only
(C) negative ion and positive ions both
P Q
(D) electrons and positive ions both

2. When a potential difference (V) is applied across a


conductor , the thermal speed of electrons is (A) vp = vQ (B) vp < vQ
(A) zero (C) vp > vQ (D) Data insufficient
(B) proportional to T
(C) proportional to (T) 8. A current (I) flows through a uniform wire of
(D) proportional to V diameter (d) when the mean drift velocity is V. The
same current will flow through a wire of diameter
3. A steady current is passing through a linear
d/2 made of the same material if the mean drift
conductor of non-uniform cross-section. The
current density in the conductor is velocity of the electron is
(A) independent of area of cross-section (A) V/4 (B) V/2
(B) directly proportional to area of cross-section (C) 4V (D) 2V
(C) inversely proportional to area of cross-section
(D) inversely proportional to the square root of
Section B - Resistance & resistivity, Ohm's
area of cross-section
law + VI (Volt - Curr.)
4. Two wires each of radius of cross section r but of Characteristics + Colour Code
different materials are connected together end to + Temp. Dependence
end (in series). If the densities of charge carriers in
the two wires are in the ratio 1 : 4, the drift velocity
9. A carbon resistor is marked with the rings coloured brown,
of electrons in the two wires will be in the ratio.
black, green and gold. The resistance (in ohm) is -
(A) 1 : 2 (B) 2 : 1
(A) 3.2 × 105 ± 5 % (B) 1 × 106 ± 10 %
(C) 4 : 1 (D) 1 : 4 7
(C) 1 × 10 ± 5 % (D) 1 × 106 ± 5 %
5. In a wire of cross-section radius r, free electrons
travel with drift velocity v when a current  flows 10. The resistance of a semi-conductors
through the wire. What is the current in another (A) increases with increase of temperature
wire of half the radius and of the same material
(B) decreases with increase of temperature
when the drift velocity is 2v ?
(A) 2I (B) I (C) does not charge with charge of temperature
(C) I/2 (D) I/4 (D) first decreases and then increases with increase
of temperature
6. An insulating pipe of cross-section area ‘A’ contains
an electrolyte which has two types of ions  their
11. A hollow copper tube of 5 m length has got
charges being –e and +2e. A potential difference
applied between the ends of the pipe result in the external diameter equal to 10 cm and the walls are
drifting of the two types of ions, having drift speed = v 5 mm thick. If specific resistance of copper is
(–ve ion) and v/4 (+ ve ion). Both ions have the same 1.7 × 10-8 ohm × metre. calculate the resistance of
number per unit volume = n. The current flowing the tube.
through the pipe is (A) 5.77 × 105  (B) 5.77 × 10-5 
(A) nev A/2 (B) nev A/4
(C) 5.77 × 10-7  (D) 5.77 × 107 
(C) 5nev A/2 (D) 3nev A/2
12. In order to increase the resistance of a given wire 16. A rectangular carbon block has dimensions
of uniform cross section to four times its value, a 1.0 cm × 1.0 cm × 50 cm. Resistances are
fraction of its length is stretched uniformly till the measured, first across two square ends and then
across two rectangular ends, respectively. If
3
full length of the wire becomes times the original resistivity of carbon is 3.5 × 10–5 -m, then values
2
of measured resistances respectively are :
length what is the value of this fraction ?
1 1
(A) (B)
4 8

1 1
(C) (D)
16 6 50 cm
1cm
13. A conductor with rectangular cross section has
dimensions (a × 2a × 4a) as shown in figure. 1cm
Resistance across AB is x, across CD is y and
across EF is z. Then 35
(A)  10 2 , 7  10  5 
2
C F 5 15
(B) 7  10  ,  10  2 
2
2a 35
A
B (C)  10  4 , 7  10  7 
4a a 2
15
(D) , 7  10 2 
E D 2

(A) x = y = z (B) x > y > z Section C, D - Circuit theory, KCL & KVL,
(C) y > z > x (D) x > z > y Battery, Grouping of cells
17. In the circuit shown, what is the potential
14. A piece of copper and another of germanium are difference VPQ ?
cooled from room temperature to 80K. The resistance of
Q
(A) each of them increases
(B) each of them decreases 2V 4V
(C) copper increases and germanium decreases 1V
(D) copper decreases and germanium increases P

(A) + 3V (B) + 2V
15. The current in a metallic conductor is plotted (C) – 2V (D) none
against voltage at two different temperatures T1
and T2. Which is correct 18. The equivalent resistance between points A and B is :
65 A
 2 15
(A) B
T1 2
Current

T2 45
(B)  8 10
2

5
Voltage (C) 
2
20 10
(A) T1 > T2 (B) T1 < T2 91
(C) T1 = T2 (D) none (D)  30 40
2
19. Consider an infinte ladder network shown in figure. 24. In the figure shown, battery 1 has emf = 6V and
A voltage V is applied between the points A and B. internal resistance = 1. Battery 2 has emf = 2V
This applied value of voltage is halved after each and internal resistance = 3  . The wires have
section. negligible resistance. What is the potential
difference across the terminals of battery 2?
R1 R1 R1 R1 R1
A
R2 R2 R2 R2 R2 1
1
R
(A) R1/R2 = 1 (B) R1/R2 = 1/2 3
(C) R1/R2 = 2 (D) R1/R2 = 3
2
20. In the diagram resistance between any two
junctions is R. Equivalent resistance across terminals (A) 4V (B) 1.5V
A and B is (C) 5V (D) 0.5V

r r

25.  i i 
(a) (b)

A B r r
 
11R 18R (No current) (d)
(A) (B)
7 11 (c)

7R 11R In which of the above cells, the potential difference


(C) (D)
11 18 between the terminals of a cell exceeds its emf.
(A) a (B) b
21. A storage battery is connected to a charger for (C) c (D) d
charging with a voltage of 12.5 Volts. The internal
resistance of the storage battery is 1 . When the 26. The diagram besides shows a circuit used in an
charging current is 0.5 A, the emf of the storage experiment to determine the emf and internal
battery is : resistance of the cell C. A graph was plotted of the
(A) 13 Volts (B) 12.5 Volts potential difference V between the terminals of the
(C) 12 Volts (D)11.5 Volts cell against the current I, which was varied by
adjusting the rheostat. The graph is shown on the right;
22. The terminal voltage across a battery of emf E can be x and y are the intercepts of the graph with the axes
(A) 0 (B) > E as shown. What is the internal resistance of the cell ?
(C) < E (D) all of above
V(V)
23. One end of a Nichrome wire of length 2L and cross- C
sectional area A is attatched to an end of
another Nichrome wire of length L and cross-sectional V
area 2A. If the free end of the longer wire is at an
electric potential of 8.0 volts, and the free end of the A
shorter wire is at an electric potential of 1.0 volt, the I(A)
potential at the junction of the two wires is equal to
(A) 2.4 V (B) 3.2 V (A) x (B) y
(C) 4.5 V (D) 5.6 V (C) x/y (D) y/x
27. A cell of emf E has an internal resistance r & is P R Q

connected to rheostat. When resistance R of R


rheostat is changed correct graph of potential
difference across it is R R
R

V V
(A) 7 (B) 4 (C) 2 (D) 5
(A) (B)
R R 31. For the following circuits, the potential
difference between X and Y in volt is –

V V

(C) (D)
R R

28. A circuit is comprised of eight identical batteries


and a resistor R = 0.8. Each battery has an emf 2 4 8 5
(A) (B) (C) (D)
of 1.0 V and internal resistance of 0.2. The voltage 3 3 9 3
difference across any of the battery is
32. In a closed circuit the sum of total emf is
equal to the sum of the –
(A) currents
(B) resistances
(C) products of current and the resistances
(D) none of the above

(A) 0.5 V (B) 1.0 V


(C) 0 V (D) 2 V Section E - Electrical Power & Energy
33. A line having a total resistance of 0.5 deliver 15kW
29. Two batteries one of the emf 3V, internal at 240 V to a small factory. The efficiency of
resistance 1 ohm and the other of emf 15 V, internal transmission is:
resistance 2 ohm are connected in series with a (A) 60% (B) 79%
resistance R as shown. If the potential difference (C) 89 % (D) 97%
between a and b is zero the resistance of R in ohm is
34. The battery in the diagram is to be charged by the
a b generator G. The generator has a terminal
voltage of 120 volts when the charging current is
3V,1 15V,2
10 amperes. The battery has an emf of 100 volts
and an internal resistance of 1 ohm. In order to
R charge the battery at 10 amperes charging current,
the resistance R should be set at
(A) 5 (B) 7
R
(C) 3 (D) 1
+
G
30. The equivalent resistance between the –
– +
terminal point P and Q is 4 in the given
100 V,1
circuit, then find out the resistance of R in
ohms – (A) 0.1 (B) 0.5 
(C) 1.0  (D) 5.0 
35. A resistor of resistance R is connected to a cell of 41. If the length of the filament of a heater is reduced
internal resistane 5 . The value of R is varied from by 10%, the power of the heater will
1  to 5 . The power consumed by R : (A) increase by about 9%
(A) increases continuously (B) increase by about 11%
(B) decreases continuously (C) increase by about 19%
(C) first decreases then increases (D) decrease by about 10%
(D) first increases then decreases.
42. A heater A gives out 300 W of heat when connected
36. Power generated across a uniform wire connected to a 200 V d.c. supply. A second heater B gives out
across a supply is H. If the wire is cut into n equal 600 W when connected to a 200 v d.c. supply. If a
parts and all the parts are connected in parallel series combination of the two heaters is connected
across the same supply, the total power generated to a 200 V d.c. supply the heat output will be
in the wire is (A) 100 W (B) 450 W
H (C) 300 W (D) 200 W
(A) (B) n2H
n2

(C) nH (D)
H Section F - Electrical Instrument + Exp.
n Verifying ohm's law & Specific
Resistance Using Meter Brige &
37. A constant voltage is applied between the two ends Post Office + Potentiometer
of a uniform metallic wire. Some heat is developed (EMF & Int. Res.)
in it. The heat developed is doubled if 43. If an ammeter is to be used in place of a voltmeter
(A) both the length and the radius of the wire are halved then we must connect with the ammeter a
(B) both the length and the radius of the wire are doubled (A) Low resistance in parallel
(C) the radius of the wire is doubled (B) High resistance in parallel
(D) the length of the wire is doubled (C) High resistance in series
(D) Low resistance in series
38. When electric bulbs of same power, but different
marked voltage are connected in series across the
44. When an ammeter of negligible internal resistance is
power line, their brightness will be
inserted in series with circuit it reads 1A. When the
(A) proportional to their marked voltage
voltmeter of very large resistance is connected across
(B) inversely proportional to their marked voltage
X it reads 1V. When the point A and B are shorted
(C) proportional to the square of their marked voltage
by a conducting wire, the voltmeter measures 10 V
(D) inversely proportional to the square of their
across the battery. The internal resistance of the
marked voltage
battery is equal to
(A) zero x
39. Rate of dissipation of Joule’s heat in resistance per
(B) 0.5  y
unit volume is (symbols have usual meaning)
(C) 0.2 
(A) E (B) J
(D) 0.1 
(C) JE (D) None 12V
45. Resistances R1 and R2 each 60 are connected in
40. The charge flowing through a resistance R varies series as shown in figure. The Potential difference
with time as Q = 2t – 8t2. The total heat produced between A and B is kept 120 volt. Then what will
1 be the reading of voltmeter connected between the
in the resistance is (for 0  t  ) point C & D if resistance A B
8
of voltmeter is 120.
R R (A) 48 V
(A) joules (B) joules
6 3 (B) 24 V R1 C R2
(C) 40 V
R D
(C) joules (D) R joules (D) None V
2
46. By error, a student places moving-coil voltmeter V 50. An ammeter A of finite resistance, and a
(nearly ideal) in series with the resistance in a resistor R are joined in series to an ideal cell C. A
circuit in order to read the current, as shown. The potentiometer P is joined in parallel to R. The
voltmeter reading will be ammeter reading is I0 and the potentiometer reading
(A) 0 E=12V, r  2 is V0. P is now replaced by a voltmeter of finite
(B) 4V resistance.The ammeter reading
(C) 6V C
4 now is I and the voltmeter
(D) 12V V eading is V.
(A) I > I0, V < V0 R
47. In the figure shown for gives values of R1 and R2 A
the balance point for Jockey is at 40 cm from A. (B) I > I0, V = V0
When R2 is shunted by a resistance of 10 , balance (C) I = I0, V < V0
(D) I < I0, V = V0 P
shifts to 50 cm. R1 and R2 are (AB = 1m)
R1 R2
10
(A) , 5 51. A battery of emf E0 = 12 V is connected across a
3 G
(B) 20, 30 4m long uniform wire having resistance 4/m. The
(C) 10, 15 cells of small emfs 1 = 2V and 2 = 4V having
A B
internal resistance 2  and 6  respectively, are
15
(D) 5,  connected as shown in the figure. If galvanometer
2
shows no deflection at the point N, the distance of
48. A 6V battery of negligible internal resistance is point N from the point A is equal to
0 R  8
connected across a uniform wire of length 1m. The 1
(A) m
positive terminal of another battery of emf 4V and 6
internal resistance 1 is joined to the point A as N
shown in figure. The ammeter shows zero deflection 1 A
(B) m B
when the jockey touches 6V 3 1 r1 G
the wire at the point C. (C) 25 cm
The AC is equal to 2 r2
(D) 50 cm
(A) 2/3 m C
A B
(B) 1/3 m 52. A potentiometer wire has length 10 m and resistance
(C) 3/5 m 10. It is connected to a battery of EMF 11 volt
A
(D) 1/2 m and internal resistance 1, then the potential gradient
4 V,1
in the wire is
49. The figure shows a metre- bridge circuit, with
(A) 10 V/m (B) 1 V/m
AB = 100 cm, X = 12 and R = 18, and the
jockey J in the position of balance. If R is now made (C) 0.1 V/m (D) none
8 , through what distance will J have to be moved
to obtain balance ? 53. In the circuit shown in figure reading of voltmeter is
– +
(A) 10 cm V1 when only S1 is closed, reading of voltmeter is
(B) 20 cm V2 when only S2 is closed. The reading of voltmeter
X R
(C) 30 cm is V3 when both S1 and 3R
A J B R
(D) 40 cm S2 are closed then
6R S1
(A) V2 > V1 > V3
S2
(B) V3 > V2 > V1 V
(C) V3 > V1 > V2
(D) V1 > V2 > V3 E
Exercise - 2 (Level-I) Objective Problems | JEE Main
Section A - Definition of Current, Current Section B - Resistance & resistivity, Ohm's
Density, Drift Velocity law + VI (Volt - Curr.)
Characteristics + Colour Code
1. The electric current in a liquid is due to the flow of
+ Temp. Dependence
(A) electron only
(B) positive ions only 7. A negligibly small current is passed through a wire
of length 15 m and uniform cross-section 6.0×10-7
(C) negative and positive ions both
m2. The measured resistance of the wire is 5.0 .
(D) electrons and positive ions both
The resistivity of the material is
(A) 1.8×10-6 -m (B) 0.2×10-6 -m
2. A metallic block has no potential difference applied
(C) 0.6×10-6 -m (D). 0.9 ×10-6 -m
across it. Then the mean velocity of free electron is
(A) proportional to T 8. The resistance of a wire of length 20 cm is 5. It is
(B) proportional to T stretched uniformly to a length of 40 cm. The
(C) zero resistance now becomes:
(D) finite but independent of temperature (A) 5 (B) 10
(C) 20 (D) 200
3. Which of the following quantities do not change 9. Read the following statements carefully :
when a resistor connected to a battery is heated due Y : The resistivity of a semiconductor decreases with
to the current?
increases of temperature.
(A) drift speed Z : In a conducting solid, the rate of collision between
(B) resistivity free electrons and ions increases with increase of
(C) resistance temperature.
(D) number of free electrons. Select the correct statement from the following
(A) Y is true but Z is false
4. Two wires of the same material having radii in the (B) Y is false but Z is true
ratio 1:2, carry currents in the ratio 4 : 1. The ratio (C) Both Y and Z are true
of drift velocities of electrons in them is: (D) Y is true and Z is the correct reason for Y
(A) 1:16 (B) 16:1
(C) 1:4 (D) 4:1 10. A brass disc and a carbon disc of same radius are
assembled alternatively to make a cylindrical
conductor. The resistance of the cylinder is
5. A uniform copper wire carries a current i amperes
independent of the temperature. The ratio of
and has p carriers per metre3. The length of the
wire is  metres and its cross-section area is s metre2. thickness of the brass disc to that of the carbon disc
If the charge on a carrier is q coulombs, the drift is [ is temperature coefficient of resistance &
velocity in ms–1 is given by Neglect linear expansion]
(A) i/sq (B) i/psq  C C  C B
(C) psq/i (D) i/ps  q (A)   (B)  
B B B C


6. In the presence of an applied electric field (E ) in a B  C B B
(C)   (D)  
metallic conductor. C B C C

(A) The electrons move in the direction of E
 11. Current density in a cylindrical wire of radius R is
(B) The electrons move in a direction opposite to E
(C) The electrons may move in any direction  x  R
 J0   1 for 0  x 
randomly, but slowly drift in the direction of E  R 2
given as J   . The current
(D) The electrons move randomly but slowly drift J x R
0 for  x  R
  R 2
in a direction opposite to E
flowing in the wire is
7 1 Section C, D - Circuit theory, KCL & KVL,
(A) J0 R 2 (B) J0 R2 Battery, Grouping of cells
24 6
18. ABCD is a square where each side is a uniform wire
7 5
(C) J0 R2 (D) J0 R2 of resistance 1. A point E lies on CD such that if a
12 12 uniform wire of resistance 1 is connected across
AE and constant potential difference is applied across
12. Specific resistance of a wire depends on the A and C then B and E are equipotential.
(A) length of the wire 1
CE A B
(B) area of cross–section of the wire (A) 1
ED 1
(C) resistance of the wire 1 1
(D) material of the wire CE
(B) 2 E
ED D C
1
13. A cross–sectional area of a copper wire is CE 1 CE
3×10–6 m2. The current of 4.2 amp. is flowing (C) ED  (D)  2
2 ED
through it. The current density in amp/m 2
through the wire is – 19. The resistance of all the wires between any two
(A) 1.4 × 103 (B) 1.4 × 104 adjacent dots is R. Then equivalent resistance
(C) 1.4 × 105 (D) 1.4 × 106 between A and B as shown in figure is :
A
(A) 7/3 R
14. When the resistance wire is passed through (B) 7/6 R
a die the cross–section area decreases by 1%, (C) 14/8 R
the change in resistance of the wire is (D) None of these B
(A) 1% decrease (B) 1% increase
(C) 2% decrease (D) 2% increase
20. In the box shown current i enters at H and leaves at
i 2i i i i
C. If iAB  , iDC  , iHA  , iGF  ,iHE  ,
15. In the following diagram two parallelopiped 6 3 2 6 6
A and B are of the same thickness. The arm choose the branch in which current is zero
of B is double that of A. Compare these resistances
(A) BG B C
and find out the value of RA/RB is –
(A) 1 (B) FC A
D
(B) 2
i i i (C) ED G
A F
1 B H
a 2a
(C) (D) none i E
2 t a 21. Consider the circuit shown in the figure
(D) 4 t
2a
5 3 A 3
16. A cylindrical copper rod is reformed to twice
its original length with no change in volume.
The resitance between its ends before the 28 V 10  10  4
change was (R). Now its resistance
(A) 8R (B) 6R
(C) 4R (D) 2R 4  B 2 3

17. Three copper wires of lengths and cross- (A) the current in the 5  resistor is 2A
(B) the current in the 5 resistor is 1A
 A
sectional areas are (  , A);  2 ,  and (C) the potential difference VA – VB is 10 V
 2 (D) the potential difference VA – VB is 5V
 
 2 , 2 A  Resistance is minimum is :– 22. The equivalent resistance of a group of
 
resistances is R. If another resistance is connected
A in parallel to the group, its new equivalent becomes
(A) wire of cross-sectional are
2 R1 & if it is connected in series to the group, its new
(B) wire of cross-sectional are A equivalent becomes R2 we have
(C) wire of cross-sectional area 2A (A) R1 > R (B) R1 < R
(D) same is all the three cases (C) R2 > R (D) R2 < R
23. The effective resistance between the points P and Which of the graphs given below represents current
Q of the electrical circuit shown in the figure is voltage characteristics when P and Q are in series.
(A) 2 Rr / (R + r) 2R 2R
2 2
(B) 8R(R + r)/(3R + r) 2R
1 1
(C) 2r + 4R P r r Q
2R (A) (B)
(D) 5R/2 + 2r 10 20 10 20
2R 2R P.D.(Volt) P.D.(Volt)

24. A wire of length L and 3 identical cells of negligible


2 2
internal resistances are connected is series. Due to
the current, the temperature of the wire is raised by 1 1
T in time t. N number of similar cells is now (C) (D)
10 20 10 20
connected in series with a wire of the same material P.D.(Volt) P.D.(Volt)
and cross section but of length 2L. The temperature
of the wire is raised by the same amount T in the 29. Find the current flowing through the resistance R1
same time t. The value of N is : of the circuit shown in figure if the resistance are
(A) 4 (B) 6 equal to R1 = 10 , R2 =20 , and R3 = 30 , and
(C) 8 (D) 9 the potential of points 1, 2 and 3 are equal to
1  10 V,  2  6 V and 3  5 V .
25. Under what condition current passing through the
resistance R can be increased by short circuiting the 2
R2
battery of emf E2. The internal resistances of the
two batteries are r1 and 1 R1 O
E1 r1 E2 r2
r2 respectively.
R2
(A) E2r1 > E1(R + r2)
(B) E1r2 > E2(R + r1) 3
(C) E2r2 > E1(R + r2) (A) 0.1 A (B) 0.2 A
R
(D) E1r1 > E2 (R + r1) (C) 0.3 A (D) 0.4 A

26. n identical cells are joined in series with its two cells 30. A battery of emf E and internal resistance r is
A and B in the loop with reversed polarities. EMF connected across a resistance R. Resistance R can be
of each shell is E and internal resistance r. Potential adjusted to any value greater than or equal to zero. A
difference across cell A or B is (here n > 4) graph is plotted between the current (i)
passing through the resistance and potential differ ence
2E  1
(A) (B) 2E 1 – n  (V) across it. Select the correct alternative (s).
n
V
4E  2 (volt)
(C) (D) 2E1  n  10
n

27. In the network shown the potential difference i


2 ampere
between A and B is (R = r1 = r2 = r3 = 1 , E1 = 3 V,
E2 = 2 V, E3 = 1 V) r E 1 1 (A) internal resistance of battery is 5
(A) 1 V (B) emf of the battery is 20V
R r E
(B) 2 V A
2 2
(C) maximum current which can be taken from the
(C) 3 V B battery is 4A
(D) 4 V (D) V-i graph can never be a straight line as shown
r 3 E3
in figure.
28. Two current elements P and Q have current 31. A simple circuit contains an ideal battery and a
voltage characteristics as shown below : resistance R. If a second resistor is placed in parallel
with the first.
P Q
(A) the potential across R will decrease
1
(B) the current through R will decreased
1
(C) the current delivered by the battery will increase
10 P.D. (Volt) 10 P.D. (Volt) (D) the power dissipated by R will increased.
32. In previous problem, if the cell had been 36. The ratio of powers dissipatted respectively in R
connected in parallel (instead of in series) which of and 3R, as shown is R
the above graphs would have shown the relationship (A) 9
between total current I and n ?
(B) 27/4
3R
(C) 4/9
I/A

I/A
(D) 4/27 2R
(A) (B)
n n
O O 37. In the circuit shown, the resistances are given in ohms
and the battery is assumed ideal with emf equal to 3.0
volts The resistor that dissipates the most power is
50
I/A

I/A

(C) (D)
R1 R3 R4 30
n n 60
O O 3V R2 50

Section E - Electrical Power & Energy


(A) R1 (B) R2
33. Arrange the order of power dissipated in the given (C) R3 (D) R4
circuits, if the same current is passing through all
circuit and each resistor is ‘r’ : 38. The variation of current (I) and voltage (V) is as shown
in figure A. The variation of power P with
current I is best shown by which of the following graph
(I) A B
V

(ii) A B
Fig A I
(iii) A P P
B

(A) (B)

(iv) A I I
B
P P

(A) P2 > P3 > P4 > P1 (B) P3 > P2 > P4 > P1


(C) (D)
(C) P4 > P3 > P2 > P1 (D) P1 > P2 > P3 > P4
I I
34. Two bulbs one of 200 volts, 60 watts & the other of 39. If X, Y and Z in figure are identical lamps, which of
200 volts, 100 watts are connected in series to a the following changes to the brightnesses of the
200 volt supply. The power consumed will be lamps occur when switch S is closed ?
(A) 37.5 watt (B) 160 watt
z
(C) 62.5 watt (D) 110 watt x s

35. Three 60 W light bulbs are mistakenly wired in series


and connected to a 120 V power supply. Assume
the light bulbs are rated for single connection to 120 Y
V. With the mistaken connection, the power dissipated (A) X stays the same, Y decreases
by each bulb is (B) X increases, Y decreases
(A) 6.7 W (B) 13.3 W (C) X increases, Y stays the same
(C) 20 W (D) 40 W (D) X decreases, Y increases
40. What amount of heat will be generated in a coil of 44. A galvanometer may be converted into ammeter or
resistance R due to a charge q passing through it if voltmeter. In which of the following cases the
the current in the coil decreases to zero uniformly resistance of the device will be the largest ? (Assume
during a time interval t maximum range of galvanometer = 1mA)
(A) an ammeter of range 10A
4 q2R q2R
(A) (B) ln (B) a voltmeter of range 5 V
3 t 2t (C) an ammeter of range 5A
2q2R ( 2  t) (D) a voltmeter of range 10 V.
(C) (D) ln
3 t q2R
45. In the circuit shown the readings of ammeter and
Section F - Electrical Instrument + Exp. voltmeter are 4A and 20V respectively. The meters
Verifying ohm's law & Specific are non ideal, then R is :
Resistance Using Meter Brige & R
Post Office + Potentiometer A

(EMF & Int. Res.) V


41. In the given circuit the current flowing through the (A) 5
resistance 20 ohms is 0.3 ampere while the (B) less than 5
ammeter reads 0.8 ampere. What is the value of R1 ? (C) greater than 5
R1 (D) between 4 & 5
20
A 46. In the given potentiometer circuit, the resistance of
the potentiometer wire AB is R0. C is a cell of internal
15
resistance r. The galvanometer G does not give zero
(A) 30 ohms (B) 40 ohms deflection for any position of the Jockey J. Which
(C) 50 ohms (D)60 ohms of the following cannot be a reason for this?
D R
+ –
42. In a balanced wheat stone bridge, current in the
galvanometer is zero. It remains zero when A B
(1) battery emf is increased r J
C
(2) all resistances are increased by 10 ohms G
(3) all resistances are made five times
(4) the battery and the galvanometer are interchanged (A) r > R0
(A) only (1) is correct (B) R >> R0
(B) (1), (2) and (3) are correct (C) emf of C > emf of D
(C) (1), (3) and (4) are correct (D) The negative terminal of C is connected A.
(D) (1) and (3) are correct
47. What will the ideal ammeter reading for the circuit
43. In the arrangement shown in figure when the switch shown here ?
S 2 is open, the galvanometer shows no
deflection for l = L/2. When the switch S2 is closed, 90
+ A
the galvanometer shows no deflection for l = 5L/

12. The internal resistance (r) of 6 V cell, and the
emf E of the other battery are respectively. 10
10

10  20
( )

6V S2
r
G
l
(A) 0.5A (B) 45/130 A
A B (C) 45/94A (D) 9/23A
L
( )
E S1

(A) 3, 8V (B) 2, 12V


(C) 2, 24V (D) 3, 12V
Exercise - 2 (Level-II) Multiple Correct | JEE Advanced

Section A - Definition of Current, Current 4. In the circuit shown E, F, G and H are cells of e.m.f.
Density, Drift Velocity 2V, 1V, 3V and 1V respectively and their
1. A current passes through a wire of nonuniform cross internal resistances are 2  , 1  , 3  and 1 
section. Which of the following quantities are respectively.
independent of the cross-section ? A
(A) the charge crossing in a given time interval. F E
(B) drift speed
D B
(C) current density 2
(D) free-electron density G H
C
Section B - Resistance & resistivity, Ohm's (A) VD – VB = –2/13 V
law + VI (Volt - Curr.)
(B) VD – VB = 2/13 V
Characteristics + Colour Code
+ Temp. Dependence (C) VG = 21/13 V = potential difference across G.
(D) VH = 19/13 V = potential difference across H.
2. Square copper plates of equal thickness and having,
area 1 cm2 and 1 m2, are connected in series as Section E - Electrical Power & Energy
shown in the Figure.
5. A metallic conductor of irregular cross-section is as
shown in the figure. A constant potential difference is
applied across the ends (1) and (2). Then :

(1) ×P ×Q (2)

(A) the current at the cross-section P equals the


(A) The resistance of the both the plates are equal current at the cross-section Q
(B) the electric field intensity at P is less than that at Q.
(B) The potential drop across both the plates are equal
(C) the rate of heat generated per unit time at Q is
(C) The current in both the plates are equal
greater than that at P
(D) The total heat dissipated in both the plates per (D) the number of electrons crossing per unit area
unit time are equal. of cross-section at P is less than that at Q.

Section C, D - Circuit theory, KCL & KVL, 6. The value of the resistance R in figure is adjusted
Battery, Grouping of cells such that power dissipated in the 2 resistor is
3. A battery is of emf E is being charged from a charger maximum. Under this condition
such that positive terminal of the battery is connected 8
to terminal A of charger and negative
terminal of the battery is connected to terminal B of
12V
charger. The internal resistance of the battery is r. R 2
(A) Potential difference across points A and B must
be more than E
(B) A must be at higher potential than B (A) R = 0
(B) R = 8
(C) In battery, current flows from positive terminal
(C) power dissipated in the 2 resistor is 72 W
to the negative terminal
(D) power dissipated in the 2 resistor is 8 W
(D) No current flows through battery
7. A bulb is connected to a battery of emf 10 V so 9. A micrometer has a resistance of 100 and a full
that the resulting current is 10 mA. When the bulb scale range of 50A. It can be used as a voltmeter
is connected to 220 V mains, the current is 50 mA. or a higher range ammeter provided a resistance is
Choose the correct alternative (s) added to it. Pick the correct range and resistance
(A) In the first case, the resistance of the bulb is combination(s).
1k and in second case, it is 4.4 k (A) 50 V range with 10 kresistance in series
(B) It is not possible since ohm’s law is not followed (B) 10 V range with 200 k resistance in series
(C) the increase in resistance is due to heating of the (C) 5 mA range with 1  resistance in parallel
filament of the bulb when it is connected to 220 V (D) 10 mA range with 1 k resistance in parallel.
mains
(D) None of these 10. In a potentiometer arrangement. E 1 is the cell
establishing current in primary circuit. E2 is the cell
Section F - Electrical Instrument + Exp. to be measured. AB is the potentiometer wire and
Verifying ohm's law & Specific G is a galvanometer. Which of the following are
Resistance Using Meter Brige &
Post Office + Potentiometer the essential condition for balance to be obtained.
(EMF & Int. Res.) (A) The emf of E1 must be greater than the emf of E2
8. Mark out the correct options. (B) Either the positive terminals of both E1 and E2
(A) An ammeter should have small resistance. or the negative terminals of both E1 and E2 must be
(B) An ammeter should have large resistance. joined to one end of potentiometer wire.
(C) A voltmeter should have small resistance. (C) The positive terminals of E1 and E2 must be
(D) A voltmeter should have large resistance. joined to one end of potentiometer wire.
(D) The resistance of G must be less than the
resistance of AB.
Exercise - 3 | Level-I Subjective | JEE Advanced

Section A - Definition of Current, Current 9. Two long straight cylindrical conductors with
Density, Drift Velocity resistivities 1 and 2 respectively are joined
1. A current of 0.50 ampere is passing through a together as shown in figure. The radius of each of
CuSO4 solution. How many Cu ++ ions will be the conductor is a. If a uniform total current I flows
deposited on cathode in 10 seconds ? through the conductors, the magnitude of the total
free charge at the interface of the two conductor is
2. A current of 5 ampere is passing through a metallic
wire of cross-sectional area 4×10 –6 m 2 . I 1 2 I
If the density of the charge carries in the wire is
5 × 1026/m3, find the drift speed of the electrons.

Section B - Resistance & resistivity, Ohm's 10. The temperature coefficient of resistance
law + VI (Volt - Curr.) of a wire is 0.00125 per degree celcius. At 300 K
Characteristics + Colour Code + its resistance is 1 ohm. The resistance of the wire
Temp. Dependence
will be 2 ohm at what temperature :–
3. A copper wire of radius 0.1 mm and resistance 1
k is connected across a power supply of 20 V. (a)
How many electrons are transferred per second Section C, D - Circuit theory, KCL & KVL,
between the supply and the wire at one end ? (b) Battery, Grouping of cells
Write down the current density in the wire.
11. A network of nine conductors connects six points
4. A wire has a length of 2.0 m and a resistance of 5.0 A, B, C, D, E and F as shown in figure. The figure
 . Find the electric field existing inside the wire if it denotes resistances in ohms. Find the equivalent
carries a current of 10 A. resistance between A and D.

5. A metal wire of resistance R is cut into three equal


A
pieces that are then connected side by side to form 2
a new wire, the length of which is equal to one third 1 1 D 1
of the original length. The resistance of this new 1
wire is :– E F
2 1 2
6. Masses of three wires of same metal are in the
B C
1
ratio 1:2:3 and their lengths in the ratio 3:2:1.
Electrical resistance of these wires will be in the 12. (a) A car has a fresh storage battery of emf 12 V and
ratio of :–
internal resistance 5.0 × 10–2 . If the starter draws
a current of 90 A, what is the terminal voltage of
7. For two wires A and B of same material and of
same mass, the radius of A is double that of B. If the battery when the starter is on ?
the resistance of wire A is 34  then that of B will be :– (b) After long use, the internal resistance of the storage
battery increases to 500 . What maximum current
(Question No. 8 to 9) can be drawn from the battery ? Assume the emf
8. The resistances of an iron wire and a copper wire
of the battery to remains unchanged.
at 20ºC are 3.9  and 4.1  respectively. At what
temperature will the resistances be equal ? (c) If the discharged battery is charged by an external
Temperature coefficient of resistivity for iron is 5.0 emf source, is the terminal voltage of the battery
× 10 –3 K–1 and for copper it is 4.0 × 10 –3 K–1. during charging greater of less than its emf 12 V ?
Neglect any thermal expansion.
13. Find the equivalent resistance 18. For what value of R in circuit, current through 4
of the circuit between points resistance is zero.
A and B shown in figure is :
A B
(each branch is of resistance R
= 1)
4V 6V
10V
14. In the circuit shown in figure, all wires have equal
resistance r. Find the equivalent resistance between
A and B.
C
E 19. If the switches S1, S2 and S3 in the figure are arranged
such that current through the battery is minimum,
B
D find the voltage across points A and B.
F
6 3
A
6 A
S2
15. Find the current through the 10  resistor shown in
s1
figure. 1 9 1 1
10  3V S3
24 V
B
3 6
20. In given circuit determine.

4.5 V (a) The rate at which the chemical energy of the cell
is consumed

16. For the circuit shown in figure, determine the (b) The rate at which heat is generated inside the
unknown voltage drop V1. battery

+1V – (c) Electric power output


+2V –
(d) Which resistance consumes maximum power
+ + (e) Power dissipated across 4 resistance
V1
30 V I
– –
i1 8

i2 4
– 5V + – 3V +
i3 8

17. (a) Determine the potential difference between X and i


Y in the circuit shown in figure.
E = 6V, r  1
2
X + –
+ 4V 21. If a cell of constant E.M.F. produces the same
– 2V
3 – 3 5 amount of the heat during the same time in two in
+ 4V
Y dependent resistors R 1 and R2 , when they are
separately connected across the terminals of the cell,
one after the another, find the internal resistance of
(b) If intermediate cell has internal resistance r = 1 
the cell.
then determine the potential difference between X and Y.
Section E - Electrical Power & Energy (F) The rate at which the chemical energy of the
22. Three equal resistors connected in series across a cell is consumed or increased.
source of emf together dissipate 10 watts of power. (G) Potential difference across box.
What would be the power dissipated if the same (H) Electric power output across box.
resistors are connected in parallel across the same
source of emf ?
27. Find the resistor in which maximum heat will be
produced.
23. Find the current through 25V cell & power supplied
5 5
by 20V cell in the figure shown.
10V 5V 20V 30V
25V 6
4
10 

11
5

5

2

V
24. One kilowatt electric heater is to be used with 220
V.D.C supply.
Section F - Electrical Instrument + Exp.
(a) What is the current in the heater
Verifying ohm's law & Specific
(b) What is its resistance. Resistance Using Meter Brige &
(c) What is the power dissipated in the heater. Post Office + Potentiometer
(EMF & Int. Res.)
(d) How much heat in calories is produced per second.
(e) How many grams of water at 100ºC will be 28. In the circuit shown in figure the reading of ammeter
converted per minute into steam at 100ºC with the heater. is the same with both switches open as with both
[(latent heat of vaporisation of water = 540 cal/g)] 0closed. Then find the resistance R. (ammeter is ideal)
100 
25. The efficiency of a cell when connected to a A
resistance R is 60%. What will be its efficiency if
the external resistance is increased to six times. R 50 

26. In following diagram boxes may contain resistor or + –


battery or any other element then determine in 300  1.5 V
each case
29. The resistance of the rheostat shown in figure is 30
 . Neglecting the meter resistance, find the minimum
1A=i 1A=i and maximum currents through the ammeter as the
rheostat is varied.
10 V,1 10 V,1
(A) (B)
5.5 V
(A) E.m.f of battery
A
(B) Battery is acting as a source or load
10 
(C) Potential difference across each battery.
(D) Power input to the battery or output by the battery. 30 
(E) The rate at which heat is generated inside the battery. 20 
30. Find the current I & voltage V in the circuit shown. 34. A battery of emf 0 = 10 V is connected across a
1m long uniform wire having resistance 10/m.
5 7 Two cells of emf 1 = 2V and 2 = 4V having internal
60 V resistances 1 and 5 respectively are connected
41 7  as shown in the figure. If a galvanometer shows no
deflection at the point P, find the distance of point P
I V 0.4  from the point a.
20 V 4
8 10
2 10  0  10V
P
A B
1

31. For the circuit shown in the figure, find the voltage  1  2V G
across 10  resistor and the current passing 5
through it.
 2  4V

35. A potentiometer wire AB is 100 cm long and has a


5 10  10A 2 1 5A
current source current source total resistance of 10ohm. If the galvanometer shows
zero deflection at the position C, then find the value
of unknown resistance R :

32. A part of a circuit is shown in figure. Here reading of


ammeter is 5 ampere and voltmeter is 96V & voltmeter 10 V

resistance is 480 ohm. Then find the resistance R 40 cm


C
A B
R 5V
A G
r  1

V R

33. The ammeter shown in figure consists of a 480  36. An accumulator of emf 2 volt and negligible
coil connected in parallel to a 20  shunt. Find the internal resistance is connected across a uniform wire
reading of the ammeter. of length 10m and resistance 30. The appropriate
terminals of a cell of emf 1.5 Volt and internal
A resistance 1 is connected to one end of the wire,
10  and the other terminal of the cell is connected through
a sensitive galvanometer to a slider on the wire. What
length of the wire will be required to produce zero
deflection of the galvanometer ? How will the
20 V balancing change (a) when a coil of resistance 5 is
placed in series with the accumulator, (b) the cell of
1.5 volt is shunted with 5 resistor ?
Exercise - 3 | Level-II Subjective | JEE Advanced

Section A - Definition of Current, Current Section C, D - Circuit theory, KCL & KVL,
Density, Drift Velocity Battery, Grouping of cells
1. The area of cross-section, length and density of a 6. A triangle is constructed using the wires AB, BC &
piece of a metal of atomic weight 60 are CA of same material and of resistance , 2 & 3
10–6 m2, 1.0 m and 5 × 103 kg/m3 respectively. Find respectively. Another wire of resistance /3 from
the number of free electrons per unit volume of
A can make a sliding contact with wire BC. Find
every atom contributes one free electron. Also find
the maximum resistance of the network between
the drift velocity of electrons in the metal when a
current of 16 A passes through it. points A and the point of sliding wire with BC.
(Given: Avogadro number 6 × 1023/ mol)
7. What will be the change in the resistance of a circuit
consisting of five identical conductors if two similar
Section B - Resistance & resistivity, Ohm's
conductors are added as shown by the dashed line
law + VI (Volt - Curr.)
Characteristics + Colour Code + in figure
Temp. Dependence
2. (a) The current density across a cylindrical
conductor of radius R varies according to the
 r
equation J  J0 1  R  , where r is the distance from
8. A piece of resistive wire is made up into two squares
the axis. Thus the current density is a maximum J0
with a common side of length 10 cm. A current enters
at the axis r = 0 and decreases linearly to zero at the
the rectangular system at one of the corners and
surface r = R. Calculate the current in terms of J0
leaves at the diagonally opposite corners. Show that
and the conductor’s cross sectional area is A = R2. the current in the common side is 1/5th of the
(b) Suppose that instead the current density is a maximum entering current. What length of wire connected
J0 at the surface and decreases linearly to zero at the between input and output terminals would have an
r equivalent effect.
axis so that J  J0 . Calculate the current.
R
3. A rod of length L and cross-section area A lies along 9. A network of resistance is constructed with R1 &
the x-axis between x = 0 and x = L. The R2 as shown in the figure. The potential at the points
material obeys Ohm’s law and its resistivity varies 1,2,3,.., N and V1, V2, V3,....., Vn respectively each
along the rod according to (x) = 0e–x/L. The end of having a potential k time smaller than previous one.
the rod at x = 0 is at a potential V0 and it is zero at x = L. Find
(a) Find the total resistance of the rod and the R1 R2
current in the wire. (i) R 2 and R 3 in terms of k
(b) Find the electric potential in the rod as a function of x.
V0 R1 V1 R1 V2 R1 V3 VN–1 R1 VN
4. A resistance R of thermal coefficient of resistivity
=  is connected in parallel with a resistance = 3R,
having thermal coefficient of resistivity = 2. Find V0 = kV1 R2 R2 R2 R2 R3
the value of eff.

5. The length of a given cylindrical wire is increased


by 100%. Due to the consequent decrease in
diameter the change in the resistance of the wire (ii) current that passes through the resistance R 2
will be :– nearest to the V0 in terms V0, k & R3.
10. In the circuit shown in figure, calculate the 14. A person decides to use his bath tub water to
following generate electric power to run a 40 watt bulb. The
(i) Potential difference between points a and b when bath tube is located at a height of 10m from the
switch S is open. ground & it holds 200 litres of water. If we install a
(ii) Current through S in the circuit when S is closed. water driven wheel generator on the ground, at what
rate should the water drain from the bath tube to
36v
light bulb ? How long can we keep the bulb on, if
the bath tub was full initially. The efficiency of
generator is 90%.(g = 10m/s–2)
a S b
15. The circuit shown in figure is made of a
homogeneous wire of uniform cross-section. ABCD
is a square. Find the ratio of the amounts of heat
liberated per unit time in wire A-B and C-D.
D C
11. An enquiring physics student connects a cell to a
circuit and measures the current drawn from the
cell to I1. When he joins a second identical cell is A B
series with the first, the current becomes I2. When
the cells are connected are in parallel, the current
through the circuit is I3. Show that relation between
Section F - Electrical Instrument + Exp.
the current is 3 I3 I2 = 2 I1 (I2 + I3)
Verifying ohm's law & Specific
Resistance Using Meter Brige &
12. Find the potential difference V A – VB for the Post Office + Potentiometer
circuit shown in the figure. (EMF & Int. Res.)
16. An ideal cell having a steady emf of 2 volt is
1V 1V B 1V 1V connected across the potentiometer wire of length
IV IV 10 m. The potentiometer wire is of magnesium and
having resistance of 11.5 /m. An another cell gives
a null point at 6.9 m. If a resistance of 5 is put in
A series with potentiometer wire, find the new position
1V 1V 1V 1V
of the null point.
Section E - Electrical Power & Energy 17. A galvanometer having 50 divisions provided with
13. The current I through a rod of a certain metallic a variable shunt s is used to measure the current
oxide is given by I = 0.2 V5/2, where V is the when connected in series with a resistance of 90 
potential difference across it. The rod is connected and a battery of internal resistance 10 . It is
in series with a resistance to a 6V battery of observed that when the shunt resistance are 10,
negligible internal resistance. What value should the 50, respectively the deflection are respectively 9
series resistance have so that & 30 divisions. What is the resistance of the
galvanometer ? Further if the full scale deflection
(i) the current in the circuit is 0.44
of the galvanometer movement is 300 mA, find the
(ii) the power dissipated in the rod is twice that
emf of the cell.
dissipated in the resistance.
18. A voltmeter of resistance RV and an ammeter of
resistance R A are connected in series across a
battery of emf E and of negligible internal resistance.
When a resistance R is connected in parallel to
voltmeter, reading of ammeter increases to three
times while that of voltmeter reduces to one-third.
Calculate RA and RV in terms of R.
Exercise - 4 | Level-I Previous Year | JEE Main
1. In the circuit, the galvanometer G shows zero 7. An electric bulb is rated 220 V-100 W. The power
deflection. If the batteries A and B have negligible consumed by it when operated on 110 V will be
internal resistance, the value of the resistor R will be (AIEEE 2006)
(AIEEE 2005) (A) 75 W (B) 40 W
500 (C) 25 W (D) 50 W
(A) 200  G 2V
(B) 100  A 8. The resistance of a bulb filament is 100  at a
R
(C) 500  12V B temperature of 100°C. If its temperature coefficient
of resistance be 0.005 per °C, its resistance will
(D) 1000 
become 200  at temperature of (AIEEE 2006)
2. Two sources of equal emf are connected to an (A) 300°C (B) 400°C
external resistance R. The internal resistances of the (C) 500°C (D) 200°C
two source are R 1 and R 2 (R 2 > R 1 ). If the
potential difference across the source having 9. In a Wheatstone's bridge, three resistances P, Q and
internal resistance R2, is zero, then (AIEEE 2005) R are connected in the three arms and the fourth
arm is formed by two resistances S1 and S2 connected
R2  ( R1  R2 ) in parallel. The condition for the bridge to be balanced
(A) R  (B) R  R2  R1 will be
( R2  R1 ) (AIEEE 2006)
R1 R2 R1 R2 P 2R P R(S1  S2 )
(C) R  (D) R  (A)  (B) 
( R2  R1 ) ( R2  R1 ) Q S1  S2 Q S1S2

3. A heater coil is cut into two equal parts and only P R(S1  S2 ) P R
(C)  (D) 
one part is now used in the heater. The heat Q 2S1S2 Q S1  S2
generated will now be (AIEEE 2005)
(A) doubled (B) four times
(C) one-fourth (D) halved 10. The Kirchholff's first law (i  0) and second law
(iR  E )
4. An energy source will supply a constant current into When the symbols have their usual meanings, are
the load, if its internal resistance is respectively based on- (AIEEE 2006)
(AIEEE 2005) (A) Conservation of charge, conservation of momentum
(A) equal to the resistance of the load (B) Conservation of energy, conservation of charge
(B) very large as compared to the load resistance (C) Conservation of momentum, conservation of charge
(C) zero (D) Conservation of charge, conservation of energy
(D) non-zero but less than the resistance of the load
11. The current I drawn from the 5 V source will be
5. In a potentiometer experiment the balancing with a 10
(AIEEE 2006)
cell is at length 240 cm. On shunting the cell with a
5 10 20
resistance of 2  , the balancing length becomes
120 cm. The internal resistance of the cell is
10
(AIEEE 2005) I
(A) 1 (B) 0.5  5V
+ –
(C) 4  (D) 2 
(A) 0.33 A (B) 0.5 A
6. The resistance of hot tungsten filament is about 10 (C) 0.67 A (D) 0.17 A
times the cold resistance. What will be the resistance
of 100 W and 200 V lamp, when not in use ? 12. The resistance of a wire is 5  at 50°C and 6  at
(AIEEE 2005) 100°C. The resistance of the wire at 0°C will be
(A) 40  (B) 20  (AIEEE 2007)
(A) 2  (B) 1
(C) 400  (D) 200 
(C) 4  (D) 3
13. A material B has twice the specific resistance of (ii) Calculate field E(r) at distance ‘r’ from A by
A. A circular wire made of B has twice the using Ohm’s law E = j, where j is the current per
diameter of a wire made of A. Then for the two unit area at ‘r’.
(iii) From the ‘r’ dependence of E(r), obtain the
wires to have the same resistance, the ratio lB / l A
potential V(r) at r.
of their respective lengths must be (AIEEE 2007) (iv) Repeat (i), (ii) and (iii) for current ‘I’ leaving
(A) 1 (B) 1/2 ‘D’ and superpose results for ‘A’ and ‘D’.
(C) 1/4 (D) 2
16. V measured between B and C is
14. Shown in the figure adjacent is a meter-bridge set
up with null deflection in the galvanometer. The value l l l l
(A)  (B) 
of the unknown resistor R is (AIEEE 2008) a (a  b) a (a  b)
55  R
l l l
(C)  (D)
2 a 2 (a  b) 2 (a  b)
G
17. For current entering at A, the electric field at a
20 cm distance ‘r’ from A is
1 1
(A) (B)
8r 2 r2
(A) 13.75  (B) 220 
1 1
(C) 110  (D) 55  (C) (D)
2r 2 4 r 2
15. A 5 V battery with internal resistance 2  and a 2 V 18. Thise question contains Statement-I and Statement-
battery with internal resistance 1 are connected to II. Of the four choices given after the statements,
a10 resistor as shown in the figure. (AIEEE 2008) choose the one that best describes the two statements.
P2 Statement I : The temperature dependence of
resistance is usually given as R  R0 (1  t ). The
5V 10  2V
2 1 resistance of a wire changes from 100  to 150 
when its temperature is increased from 27°C to 227°C.
P1 This implies that   2.5  103 /  C.
The current in the 10  resistor is Statement II : R  Ri (1  T ) is valid only
(A) 0.27 A, P2 to P1 (B) 0.03 A, P1 to P2
when the change in the temperature T is small
(C) 0.03 A, P2 to P1 (D) 0.27 A, P1 to P2
and R  ( R  R0 )  R0 . (AIEEE 2009)
Directions: (A) Statement I is true, Statement II is false.
Question No. 16 and 17 are based on the following (B) Statement I is true, Statement II is true; Statement
paragraph. (AIEEE 2008) II is the correct explanation of Statement I.
Consider a block of conducting material of resistivity (C) Statement I is true. Statement II is true; Statement
‘’ shown in the figure. Current ‘I’ enters at ‘A’ and II is not the correct explanation of Statement I.
leaves from ‘D’. We apply superposition principle to (D) Statement I is false, Statement II is true.
find voltage ‘V’ developed between ‘B’ and ‘C’.
The calculation is done in the following steps: 19. Two conductors have the same resistance at 0°C
but their temperature coefficients of resistance are
1 and  2 . The respective temperature coefficients
of their series and parallel combinations are nearly
(AIEEE 2010)
1   2 1   2
(A) , 1   2 (B) 1   2 ,
2 2
1 2 1   2 1   2
(C) 1   2 , (D) ,
(i) Take current ‘I’ entering from ‘A’ and assume it 1   2 2 2
to spread over a hemispherical surface in the block.
20. In the circuit shown below, the key K is closed at 26. The supply voltage to a room is 120 V. The resistance
t = 0. The current through the battery is of the lead wires is 6 A 60 W bulb is already
[AIEEE 2010] switched on. What is the decrease of voltage across
V K the bulb, when a 240 W heater is switched on in
parallel to the bulb? [AIEEE 2013]
L1 R1 (A) 13.3 Volt (B) 10.4 Volt
(C) zero Volt (D) 2.9 Volt
R2 27. This question has statement I and statement II. Of the
four choices given after the statement, choose the one
VR1R 2 that best decribes the two statements. [AIEEE 2013]
V
(A) at t = 0 and R at t =  Statement - I : Higher the range, greater is the
R12  R 22 2 resistance of ammeter.
V V(R1  R 2 )
Statement - II : To increase the range of ammeter,
(B) R at t = 0 and at t =  additional shunt needs to be used across it.
R1R 2
2 (A) Statement-I is true, Statement-II is false
VR1R 2 (B) Statement-I is false, Statement-II is true
V
(C) R at t = 0 and at t =  (C) Statement-I is true, Statement-II is true,
2 R12  R 22 Statement-II is the correct explanation of statement-I
(D) Statement-I is true, Statement-II is true, Statement-II
V(R1  R 2 ) V
(D) at t = 0 and R at t =  is not the correct explanation of Statement-I.
R1R 2 2
28. In a large building, there are 15 bulbs of 40 W, 5
bulbs 100 W, 5 fans of 80 W and 1 heater of 1 kW.
21. If 400  of resistance is made by adding four The voltage of the electric mains is 220 V. The
100  resistance of tolerance 5%, then the minimum capacity of the main fuse of the building
tolerance of the combination is (AIEEE 2011) will be : (JEE MAIN 2014)
(A) 20% (B) 5% (A) 12 A (B) 14 A (C) 8 A (D) 10 A
(C) 10% (D) 15% 29. When 5V potential difference is applied across a
wire of length 0.1 m, the drift speed of electrons is
22. The current in the primary circuit of a potentiometer 2.5 × 10–4 ms–1. If the electron density in the wire is
is 0.2 A. The specific resistance and cross-section 8 × 1028 m–3, the resistivity of the material is close
of the potentiometer wire are 4 × 10–7   m and to : (JEE MAIN 2015)
8 × 10–7 m2 respectively. The potential gradient will (A) 1.6 × 10–6 m (B) 1.6 × 10–5 m
be equal to (AIEEE 2011) (C) 1.6 × 10–8 m (D) 1.6 × 10–7 m
(A) 0.2 V/m (B) 1 V/m 30. In the circuit shown, the current in the 1 resistor is :
(C) 0.3 V/m (D) 0.1 V/m (JEE MAIN 2015)
(A) 0.13 A, from Q to P
23. If a wire is stretched to make it 0.1% longer, its
resistance will (AIEEE 2011) (B) 0.13 A, from P to Q
(A) increase by 0.2% (B) decrease by 0.2% (C) 1.3 A, from P to Q
(C) decrease by 0.05% (D) increase by 0.05%
(D) 0A
24. Two electric bulbs marked 25 W-220V and 100 W- 31. A galvanometer having a coil resistance of 100
220V are connected in series to a 440V gives a full scale deflection, when a current of
supply. Which of the bulbs will fuse? (AIEEE 2012) 1 mA is passed through it. The value of the
(A) Both (B) 100W resistance, which can convert this galvanometer into
(C) 25W (D) Neither ammeter giving a full scale deflection for a current
of 10 A, is : (JEE MAIN 2016)
25. Resistance of a given wire is obtained by mea suring (A) 2 (B) 0.1  (C) 3  (D) 0.01 
the current flowing in it and the voltage difference 32. The temperature dependence of resistances of Cu
applied across it. If the percentage errors in the and aundoped Si in the temperature range 300–400
measurement of the current and the voltage K, is best described by : (JEE MAIN 2016)
difference are 3% each, then errors in the value of (A) Linear increase for Cu, exponential increase for Si.
resistance of the wire is (AIEEE 2012) (B) Linear increase for Cu, exponential decrease for Si.
(A) 6% (B) zero (C) Linear decrease for Cu, linear decrease for Si.
(C) 1% (D) 3% (D) Linear increase for Cu, linear increase for Si.
2V 2V 2V 36. The reading of the ammeter for a silicon diode
in the given circuit is : (JEE MAIN 2018)

33.

2V 2V 2V

In the above circuit the current in each resistance


is - (JEE MAIN 2017)
(A) 0 A (B) 1 A
(C) 0.25 A (D) 0.5 A (A) 13.5 mA (B) 0
(C) 15 mA (D) 11.5 mA
34. In the given circuit diagram when the current
37. Two batteries with e.m.f. 12 V and 13 V are
reaches steady state in the circuit, the charge on connected in parallel across a load resistor
the capacitor of capacitance C wil be : of 10 . The internal resistances of the two
(JEE MAIN 2017) batteries are 1 and 2 respectively. The
voltage across the load lies between :
E r (A) 11.7 V and 11.8 V (JEE MAIN 2018)
(B) 11.6 V and 11.7 V
(C) 11.5 V and 11.6 V
r1
(D) 11.4 V and 11.5 V
C
38. On interchanging, the resistances, the
balance point of a meter bridge shifts the
r2 left by 10 cm. The resistance of there series
combination is 1 k. How much was the
resistance on the left slot before
r1
(A) CE (r  r) (B) CE interchanging the resistances ? (JEE MAIN
1 2018)
(A) 910  (B) 990 
r1 r2 (C) 505  (D) 550 
(C) CE (r  r) (D) CE (r  r )
2 2
39. In a potentiometer experiment, it is found
that no current passes through the
35. which of the following statements is false ? galvanometer when the terminals of the cell
(JEE MAIN 2017) are connected across 52 cm of the
(A) Kirchhoff’s second law represent energy con- potentiometer wire. If the cell shunted by a
resistance of 5, a balance is found when
servation. the cell is connected across 40 cm of the
(B) Wheatstone bridge is the most sensitive when wire. Find the internal resistance of the cell .
all the four resistances are of the same order mag- (JEE MAIN 2018)
nitude. (A) 25  (B) 1 
(C) In a balanced Wheatstone bridge the cell and (C) 1.5  (D) 2 
the galvanometer are exchanged, the null point is
disturbed.
(D) A rheostat can be used as a potential divider.
Exercise - 4 | Level-II Previous Year | JEE Advanced
1. A galvanometer has resistance 100  and it 5. Column-I gives some devices and Column-II
requires current 100A for full scale deflection. A gives some process on which the functioning of
resistor 0.1 is connected to make it an ammeter. these devices depend. Match the devices in
The smallest current required in the circuit to produce Column-I with the processes in Column-II and
the full scale deflection is [JEE-2005] indicate your answer by darkening appopriate
bubbles in the 4 × 4 matrix given in the ORS.
(A) 1000.1 mA (B) 1.1 mA
[JEE - 2007]
(C) 10.1 mA (D) 100.1 mA
Column-I Column-I
(A) Bimetallic strip (p) Radiation from a
2. An unknown resistance X is to be determined using hot body
resistances R1, R2 or R3. Their corresponding null points (B) Steam engine (q) Energy conversion
are A, B and C. Find which of the above will give the (C) Incandescent (r) Melting
most accurate reading and why ? [JEE-2005] lamp
(D) Electric fuse (s) Thermal expansion
of solids
(A) A - 's, q' or 'p', B - 's', C - q' or 'p' , D - 'p, r' or 'r'
(B) A - 's, r' or 's', B - 'q', C - 'p, s' or 'p' , D - 'q, p
(C) A - 's, q' or 's', B - 'q', C - 'p, q' or 'p' , D - 'q, r' or 'r'
X R (D) A - 'r, s', B - 'r', C - 'p, q' or 'p' , D - 'p, r' or 'r'

G 6. Figure shows three resistor configurations R1, R2


and R3 connected to 3V battery. If the power
A B C dissipated by the configuration R1, R2 and R3 is
R = R1 or R2 or R3 P1, P2 and P3, respectively, then [JEE - 2008]

3. Consider a cylindrical element as shown in the


3V
figure. Current flowing the through element is I and 3V
3V
resistivity of material of the cylinder is . Choose
the correct option out the following [JEE-2006]
R2
R1 R3
A B
I C (A) P1 > P2 > P3 (B) P1 > P3 > P2
4r 2r (C) P2 > P1 > P3 (D) P3 > P2 > P1

l/2 l/2 7. STATEMENT-1


In a Meter Bridge experiment, null point for an
(A) Power loss in second half is four times the power
unknown resistance is measured. Now, the unknown
loss in first half resistance is put inside an enclosure maintained at
(B) Voltage drop in first half is twice of voltage drop a higher temperature. The null point can be obtained
in second half at the same point as before by decreasing the value
(C) Current density in both halves are equal of the standard resistance. [JEE - 2008]
(D) Electric field in both halves is equal and
STATEMENT-2
4. A resistance of 2 is connected across one gap of a Resistance of a metal increases with increase in
metre-bridge (the length of the wire is 100 cm) and an temperature.
unknown resistance, greater than 2, is connected across (A) STATEMENT-1 is True, STATEMENT-2 is True;
the other gap. When these resistances are interchanged, STATEMENT-2 is a correct explanation for
STATEMENT-1
the balance point shifts by 20 cm. Neglecting any
(B) STATEMENT-1 is True, STATEMENT-2 is
corrections, the unknown resistance is
True; STATEMENT-2 is NOT a correct
[JEE-2007] explanation for STATEMENT-1
(A) 3  (B) 4  (C) STATEMENT-1 is True, STATEMENT-2 is False
(C) 5  (D) 6  (D) STATEMENT-1 is False, STATEMENT-2 is True
8. For the circuit shown in the figure 11. To verify Ohm's law, a student is provided with a
test resistor R T. a high resistance R 1 , a small
resistance R2, two identical galvanometers G1 and
G2 and a variable voltage source V. the correct circuit
to carry out the experiment is : [JEE 2010]

G1 G1

(A) the current I through the battery is 7.5 mA R2 R1


(B) the potential difference across RL is 18 V RT
G2
R1 RT
G2
R2
(A) (B)
(C) ratio of powers dissipated in R1 and R2 is 3
(D) if R1 and R2 are interchanged, magni tude of the v v

power dissipated in RL will de crease by a factor of 9


[JEE - 2009]
R1 R2
G1 G1
9. Consider a thin square sheet of side L and thickness
G2 G2
t, made of a material of resistivity . The resistance RT RT
(C) R2
(D) R1
between two opposite faces shown by the shaded
area in the figure is : [JEE 2010] v v

12. When two identical batteries of internal resistance


1  each are connected in series across a resistor R,

t the rate of heat produced in R is J1. When the same


L batteries are connected in parallel across R, the rate is
J2. If J1 = 2.25 J2 then the value of R in  is
[JEE 2010]
(A) directly proportional to L
(B) directly proportional to t
(C) independent of L 13. A meter bridge is set-up as shown, to determine an
(D) independent of t unknown resistance 'X' using a standard 10 ohm
resistor. The galvanometer shows null point when
10. Incandenscent bulbs are designed by keeping in mind tapping-key is at 52 cm mark. The end-corrections
that the resitance of their filament increases with are 1 cm and 2 cm respectively for the ends A and
the increase in temperature. If at room temperature, B. The determined value of 'X' is [JEE 2011]
100 W, 60 W and 40 W bulbs have filament
resistances R100, R60 and R40 , respectively, the X 10

relation between these resistance is [JEE 2010]

1 1 1 C
(A) R  R  R (B) R100 = R40 + R60
100 40 60 A B

1 1 1 (A) 10.2 ohm (B) 10.6 ohm


(C) R100 > R60 > R40 (D) R  R  R
100 60 40 (C) 10.8 ohm (D) 11.1 ohm
14. Two batteries of different emfs and different internal 18. A galvanometer gives full scale deflection with 0.006
resistances are connected as shown. The voltage A current. By connecting it to a 4990  resistance, it
across AB in volts is [JEE 2011]
can be converted into a voltmeter of range 0-30 V. If
6V
2n
connected to a  resistance, it becomes an ammeter
A B 246

of range 0 - 1.5 A. The value of n is [JEE 2014]

3V 2
19. In an aluminum (Al) bar of square cross section, a
15. For the resistance network shown in the figure,
square hole is drilled and is filled with iron (Fe) as
choose the correct option(s). [JEE 2012]
shown in the figure. The electrical resistivities of Al
I2  and Fe are 2.7 × 10–8 m and 1.0 × 10–7 m,

 respectively. The electrical resistance between the
 
two faces P and Q of the composite bar is
 [JEE Advanced 2015]


I1

(A) The current through PQ is zero


(B) l1 = 3A
(C) The potential at S is less than that at Q
(D) l2 = 2A

16. Two ideal batteries of emf V1 and


V2 and these resistances R1, R2 and
R3 are connected as shown in
the figure. The current in resistance
R2 would be zero if 2475 1875
(A)  (B) 
(A) V1 = V2 and R1 = R2 = R3 64 64
(B) V1 = V2 and R1 = 2R2 = R3
(C) V1 = 2V2 and 2R1 = 2R2 = R3 1875 2475
(C)  (D) 
(D) 2V1 = V2 and 2R1 = R2 = R3 [JEE 2014] 49 132

17. Heater of an electric kettle is made of a wire of lenght


L and diameter d. It takes 4 minutes to raise the 20. In the following circuit, the current through the
temperature of 0.5 kg water by 40 K. This heater is resistor R (=2) is I amperes. The value of I is -
replaced by a new heater having two wires of the same [JEE Advanced 2015]
material, each of length L and diameter 2d. The way R(=2) 1
these wires are connected is given in the options. Howe
2

8

much time in munutes will it take to raise the 2


6
temperature of the same amout of water by 40 K?
4

10
6.5V
(A) 4 if wires are in parallel [JEE 2014]
(B) 2 if wires are in series 12
(C) 1 if wires are in series
(D) 0.5 if wires are in parallel
21. An infinite line charge of uniform electric charge 23. Consider two identical galvanometers and two
density  lies along the axis of an electrically identical resistors with resistance R. If the internal
conducting infinite cylindrical shell of radius R, At of the galvanometers R C < R/2, which of the
time t = 0, the space inside the cylinder is filled with following statement(s) about any one of the
a material of permittivity  and electrical conductivity galvanometer is (are) true? [JEE Advanced 2016]
. The electrical conduction in the material follows (A) The maximum voltage range is obtained when
Ohm’s law. Which one of the following graphs best all the components are contained in series
describes the subsequent variation of the (B) The maximum voltage range is obtained when
magnitude of current density j(t) at any point in the the two resistors and one galvanometer are
material? [JEE Advanced 2016] connected in series, and the second galvanometer is
connected in parallel to the first galvanometer
j( t) j( t)
(C) The maximum current range is obtained when
all the components are connected in parallel
(A) (B) (D) The maximum current range is obtained when
(0, 0) t (0, 0) t the two galvanometers are connected in series and
the combination is connected in parallel with both
j( t) j(t) the resistors
24. In the circuit shown below, the key is pressed at
(C) (D) time t = 0. Which of the following statement (s) is
(0, 0) t (0, 0) t (are) true ? [JEE Advanced 2016]
40F 25k

22. An incandesecent bulb has a thin filament of –


V
tungstgen that is heated to high temperature by +
passing an electric current. The hot filament emits
50F 20F
black - body radiation. The filament is observed to
A
break up at random location after a sufficiently long
time of operation due to non - uniform evaporation
of tungsten from the filament. If the bulb is + –
powered at constant voltage, Which of the following Key 5V
statement(s) is(are) true? [JEE Advanced 2016]
(A) The voltmeter displays –5 V as soon as the key
(A) The temperature distribution over the
is pressed, and displays +5 after a long time
filament is uniform
(B) The voltmeter will display 0 V at time t = In 2 seconds
(B) The resistance over small sections of the filament
(C) The current in the ammeter becomes 1/e of the
decreases with time
initial value after 1 second
(C) The filament emits more light at higher band of
(D) The current in the ammeter becomes zero after
frequencies before it breaks up
a long time
(D) The filament consumes less electrical power
towards the ends of the life of the bulb

, Kota
Exercise - 1 Objective Problems | JEE Main
1. D 2. B 3. C 4. C 5. C 6. D 7. C
8. C 9. D 10. B 11. B 12. B 13. D 14. D
15. B 16. A 17. B 18. B 19. B 20. D 21. C
22. D 23. A 24. C 25. B 26. D 27. D 28. C
29. C 30. A 31. A 32. C 33. C 34. C 35. A
36. B 37. B 38. C 39. C 40. A 41. B 42. D
43. C 44. C 45. A 46. D 47. A 48. A 49. B
50. A 51. C 52. B 53. A

Exercise - 2 (Level-I) Objective Problems | JEE Main


1. C 2. C 3. D 4. B 5. B 6. D 7. B
8. C 9. C 10. A 11. D 12. D 13. D 14. D
15. A 16. C 17. C 18. D 19. B 20. B 21. A
22. C 23. A 24. B 25. B 26. D 27. B 28. C
29. B 30. A 31. C 32. A 33. A 34. A 35. A
36. D 37. A 38. B 39. B 40. A 41. D 42. C
43. B 44. D 45. C 46. A 47. A

Exercise - 2 (Level-II) Multiple Correct | JEE Advanced


1. A,D 2. A,B,C,D 3. A,B,C 4. A,C,D 5. A,B,C,D
6. A,C 7. A,C 8. A,D 9. B,C 10. A,B

Exercise - 3 | Level-I Subjective | JEE Advanced


2
1. 1.5625 × 1019 2. 1.56×10-2m/s 3. 1.25 × 1017, (b) × 106 A/m2

4. 25 V/m 5. R/9 6. 27:6:1 7. 544
8. 84.5°C 9. 0 I | 1 – 2 | 10 1127K 11. 1
12. (a) 7.5 V, (b) 24 mA(c) greater than 12 V Discharging condition

22 3r
13.  14. 15. zero 16. 19 V 17. (a) 3.7 V (b) 3.7 V
35 5
18. 1 19. 1V 20. (a) 12 W (b) 4W (c) 8 W (d) 4 (e) 4W

21. R1 R 2 22. 90 watt 23. 12A, – 20 W

50 22  11
24. (a)  4.55 A (b)  48 .4  (c) 1000 W (d) 240 cal s–1 (e) 80/3 gm min-1
11 5
25. 90%
26. (a) E = 10 V each (b) (A) act as a source and (B) act as load
(c) VA = 9V, VB = 11 V (d) PA = 9 W, PB = 11 W (e) Heat rate = 1 W each
(f) 10 watt. (g) 9V, 11 V (h) –9W, 11 W
3 3  5 .5
27. 4 28. 600 29.  0.15 A ,  0.83 A
20 20

25 5
30. I = 2.5 A, V = 3.5 volts 31. V  2.78 V, A  0. 278 A 32. 20 ohm
9 18

50
33. A 34. 46.67 cm 35. 4 ohm 36. 7.5 m, 8.75 m
73

Exercise - 3 | Level-II Subjective | JEE Advanced


1. 5 × 1028/m3, 2 × 10–3 m/s 2. (a) J0A/3; (b) 2J0 A/3

 0L  1 V0 A  e  V ( e  x /L  e 1 )
3. R 1  ; I  ; V 0
A  e  
 0L e  1 1  e 1

5 R2 3
4.  eff   5. 300% 6. (3/11) 7. 
4 R1 5

(k  1) 2
;
k  
(k  1) /k2 V0
8. 7/5 times the length of any side of the square 9. (i) (ii)
k (k  1) R3

22
10. (i) Vab = –12 V, (ii) 3 amp from b to a 11. proved 12.  Volt
9

13. (i) 10.52 ; (ii) 0.3125  14. 4/9 kg/sec, 450 sec 15. 11  6 2

8
16. 7.2 m 17. 233.3, 144V 18. [RA = R, RV = 8R]
3

Exercise - 4 | Level-I Previous Year | JEE Main


1. B 2. B 3. A 4. C 5. D 6. A 7. C
8. B 9. B 10. D 11. B 12. C 13. D 14. B
15. C 16. C 17. C 18. D 19. D 20. B 21. B
22. D 23. A 24. C 25. A 26. B 27. B 28. A
29. B 30. A 31. D 32. B 33. A 34. D 35. C
36. D 37. C 38. D 39. C

Exercise - 4 | Level-II Previous Year | JEE Advanced


1. D 2. This is true for r1 = r2; So R2 given most accurate value 3. A
4. A 5. C 6. C 7. D 8. A,D 9. C
10. D 11. C 12. 4 13. B 14. 5 15. A,B,C,D
16. A,B,D 17. B,D 18. 5 19. B 20. 1 21. A
22. C,D 23. A,C 24. A,B,C,D

You might also like